Download as pdf or txt
Download as pdf or txt
You are on page 1of 76

https://telegram.me/UPSCMaterials https://telegram.me/FreeUPSCMaterials https://telegram.

me/MaterialforExam
Mock Test 11

9 What is/are the key difference(s) between biosimilars and generics?


1. Biosimilars are unofficial versions of original “innovator” products and thus can only be
manufactured when the original product’s patent expires.
2. Biosimilars involve developing equivalent of biological entity, while generics involve developing
equivalent of a chemical entity - the Active Pharmaceutical Ingredient (API).

Which of the above is/are correct?


A. 1 only
B. 2 only
C. Both 1 and 2
D. None

Correct Answer : B

Answer Justification :

Justification: Statement 1: A biosimilar is a biologic medical product that is almost an identical


copy of an original product that is manufactured by a different company.

Biosimilars are officially approved versions of original “innovator” products and can be
manufactured when the original product’s patent expires. Reference to the innovator product is an
integral component of the approval.

Statement 2: In case of biosimilars, biological entities being somewhat different (and not as it is of
replica), every organism has to be engineered to produce the same therapeutic effect while in
generics, the copies of API can be generated.

Bio-similars differ from generics – in complexity, in the manufacturing processes and in the data
needed to demonstrate similarity for approval. The structure of Generic Simple and well-defined
whereas for Bio-similar its Complex with potential structural variations.

Regulatory procedure to get approval for biosimilars is complex as compared to that of a generic.

Learning: Need of Biosimilars in India:

Due to enormous soaring demand for generic drugs, India’s pharmaceutical producers
emerged as world market leaders in this sector and were a major business success story in
the 2000s.

In the process, Indian producers made a valuable contribution to reducing costs and to
expanding access to life-saving treatments for patients, both in emerging markets and in
developed countries.

Recently there is a wave of consolidation among pharma retailers, stiffer competition from
Chinese pharma manufacturers and an uptick in generic drug applications have combined to

7
https://telegram.me/UPSCMaterials https://telegram.me/FreeUPSCMaterials https://telegram.me/MaterialforExam
Mock Test 11

put downward pressure on drug prices.

It is imperative that India’s pharmaceutical manufacturers create new markets to restore


market confidence in their growth prospects.

Q Source:
http://www.insightsonindia.com/2018/09/07/insights-into-editorial-biosimilars-need-huge-investment
s-longer-timelines/

10 The 91st Amendment Act of 2003 is important in the history of the development of Indian polity
because
1. It gave constitutional recognition to the system of political parties.
2. It regulated the size of council of ministers based on the strength of Lok Sabha.

Which of the above is/are correct?


A. 1 only
B. 2 only
C. Both 1 and 2
D. None

Correct Answer : C

Answer Justification :

Justification: Statement 1: Till this amendment was introduced, the Constitution did not refer to
the existence of political parties, even though the entire system of governance is based on the party
system.

Through this amendment, there is a clear constitutional recognition of political parties for the first
time. The amendment brought in the Tenth Schedule provides that an MP/MLA/MLC forfeits his/her
membership of the legislature if he/she defects from the party on whose symbol he/she was elected.

Statement 2: As per the amendment, the total number of ministers, including the Prime Minister, in
the Council of Ministers shall not exceed 15% of the total strength of the Lok Sabha.

Q Source: Chapter - Parliament: Indian Polity: M Laxmikanth

11 The Cabinet Committee on Economic Affairs has approved continuation of the Centrally Sponsored
Umbrella Scheme of Integrated Development of Wildlife Habitats (CSS-IDWH) beyond the 12thPlan
period from 2017-18 to 2019-20. Consider the following about CSS-IDWH.
1. The scheme consists of recovery programmes for saving critically endangered species and habitats.
2. Financial assistance is also provided to States for relocation of communities from within protected
areas to other areas.
3. It does not cover protection of wildlife outside Protected Areas.

8
https://telegram.me/UPSCMaterials https://telegram.me/FreeUPSCMaterials https://telegram.me/MaterialforExam
Mock Test 11

Select the correct answer using the codes below.


A. 1 only
B. 1 and 2 only
C. 2 and 3 only
D. 1 and 3 only

Correct Answer : B

Answer Justification :

Justification and Learning: The Scheme consists of Centrally Sponsored Scheme of Project Tiger
(CSS-PT), Development of Wildlife Habitats (CSS-DWH) and Project Elephant (CSS-PE).

The implementation of the schemes would be done through the respective States in designated
Tiger Reserves, Protected Areas and Elephant Reserves.

The activities covered under the scheme include the staff development and capacity building,
wildlife research and evaluation, anti-poaching activities, wildlife veterinary care, addressing man-
animal conflicts and promoting eco-tourism.

S2 and S3: Financial assistance is also provided to States for relocation of communities from
within protected areas to other areas.

The scheme has following three components:

Support to Protected Areas (National Parks, Wildlife Sanctuaries, Conservation Reserves and
Community Reserves).

Protection of Wildlife Outside Protected Areas.

Recovery programmes for saving critically endangered species and habitats.

Significance and benefits of the scheme:

A total of 18 tiger range States, distributed in five landscapes of the country would be
benefitted under the Project Tiger scheme.

Besides immense environmental benefits and effective implementation of tiger conservation


inputs in and around tiger reserves

The schemes would address the human wildlife conflict effectively.

9
https://telegram.me/UPSCMaterials https://telegram.me/FreeUPSCMaterials https://telegram.me/MaterialforExam
Mock Test 11

These schemes would generate employment opportunities resulting in economic upliftment of


people.

These schemes would result in resource generation

Q Source: As mentioned above

12 With reference to Initial public offering or IPO, consider the following statements.
1. It is a process where a private company declares itself public by selling off its stocks in the general
public.
2. Any company, irrespective of the number of years from its time of establishment, its size and type of
business can declare an IPO.
3. After an IPO, the company's shares are traded in an open market.

Select the correct answer using the codes below.


A. 1 only
B. 2 and 3 only
C. 1 and 3 only
D. 1, 2 and 3

Correct Answer : D

Answer Justification :

Justification: In case a private company requires capital that is way beyond its individual ability to
generate through regular operations there are a few alternatives that the management can take up
to work out that capital.

Popular methods to do so include private investment, taking debt or a public investment through an
IPO.

Any company, irrespective of the number of years from its time of establishment, its size and type of
business it deals in can go ahead and get itself listed in the exchange to go public.

A company can raise equity capital with IPO, by issuing new shares to the public or the existing
shareholders can sell off their shares to other people without raising any fresh capital.

The company that sells its shares are known as an ‘issuer’ and does so with the help of investment
banks present in the market.

After an IPO, the company's shares are traded in an open market. Any company that decides to sell
its shares, aims to raise money for the company, which it can further use to grow and expand.

Q Source: Chapter 7: Shankar Ganesh: Indian Economy – Key Concepts

10
https://telegram.me/UPSCMaterials https://telegram.me/FreeUPSCMaterials https://telegram.me/MaterialforExam
Mock Test 11

13 The representatives of each union territory in the Rajya Sabha are

A. Nominated by the Union Territory (UT) Government


B. Indirectly elected by members of an electoral college specially constituted for the purpose.
C. Nominated by the President
D. Not represented in the house

Correct Answer : B

Answer Justification :

Learning: At present, the Rajya Sabha has 245 members. Of these, 229 members represent the
states, 4 members represent the union territories and 12 members are nominated by the president.

The representatives of each union territory in the Rajya Sabha are indirectly elected by members of
an electroral college specially constituted for the purpose. This election is also held in accordance
with the system of proportional representation by means of the single transferable vote. Out of the
seven union territories, only two (Delhi and Puducherry) have representation in Rajya Sabha. The
populations of other five union territories are too small to have any representative in the Rajya
Sabha.

Q Source: Chapter – Parliament: Indian Polity: M Laxmikanth

14 Consider the following statements.


1. Before the enactment of the Government of India Act 1919, the Governor- General of India presided
over the meetings of the Central Legislative Council.
2. The Government of India Act of 1935 instituted the office of the President of the Legislative
Assembly.

Which of the above is/are correct?


A. 1 only
B. 2 only
C. Both 1 and 2
D. None

Correct Answer : A

Answer Justification :

Justification: Statement 1: The institutions of Speaker and Deputy Speaker originated in India in
1921 under the provisions of the Government of India Act of 1919 (Montague–Chelmsford Reforms).

At that time, the Speaker and the Deputy Speaker were called the President and Deputy President
respectively and the same nomenclature continued till 1947.

Before 1921, the Governor- General of India used to preside over the meetings of the Central

11
https://telegram.me/UPSCMaterials https://telegram.me/FreeUPSCMaterials https://telegram.me/MaterialforExam
Mock Test 11

Legislative Council.

Statement 2: The GoI Act 1935 changed the nomenclatures of President and Deputy President of
the Central Legislative Assembly to the Speaker and Deputy Speaker respectively.

However, the old nomenclature continued till 1947 as the federal part of the 1935 Act was not
implemented.

Q Source: Chapter – Parliament: Indian Polity: M Laxmikanth

15 Nomadic Elephant-2018, recently in news, is a/an

A. Indo-Mongolia joint military exercise


B. Conservation festival for elephants
C. Program linking elephant corridors in tropical countries
D. Global investor summit

Correct Answer : A

Answer Justification :

Learning: Indo-Mongolia joint exercise Nomadic Elephant-2018, commenced recently at Mongolian


Armed Forces (MAF) Five Hills Training Area, Ullanbaatar, Mongolia with a brief but impressive
opening ceremony.

It an annual exercise since 2006 which is designed to strengthen the partnership between Indian
Army and Mongolian Armed Forces. The exercise will see them improve their tactical and technical
skills in joint counter insurgency and counter terrorist operations in rural and urban scenario under
United Nations mandate.

During the exercise both sides will jointly train, plan and execute a series of well developed tactical
drills for neutralisation of likely threats that may be encountered in urban warfare scenario. Experts
from both sides will also hold detailed discussions to share their experience on varied topics for
mutual benefits.

Exercise Nomadic Elephant-2018 will contribute immensely in developing mutual understanding &
respect for each others military and also facilitate in tackling the world wide phenomenon of
terrorism.

Q Source: http://pib.nic.in/PressReleseDetail.aspx?PRID=1545620 Release ID: 1545565

16 India and France have signed an implementation agreement on “MOBILISE YOUR CITY” (MYC)
programme. Consider the following about MYC.
1. It is one of the fifteen international transport initiatives of the UN Global Climate Action Agenda
(GCAA).
2. The project seeks to back 100 cities worldwide in three years, which are engaged in sustainable
urban mobility planning to reduce greenhouse gas emissions.

12
https://telegram.me/UPSCMaterials https://telegram.me/FreeUPSCMaterials https://telegram.me/MaterialforExam
Mock Test 11

Which of the above is/are correct?


A. 1 only
B. 2 only
C. Both 1 and 2
D. None

Correct Answer : C

Answer Justification :

Justification: Based on a proposal made by AFD in 2015, the European Union has agreed to
provide funds of Euro 3.5 million through the AFD to contribute to specific investments and
technical assistance components within the Mobilise Your City (MYC) programme in India.

MobiliseYourCity (MYC) is a global climate initiative for integrated urban mobility planning, and
one of 15 international transport initiatives of the UN Global Climate Action Agenda (GCAA).

MYC is part of an international initiative which is supported by the French and the German
Governments and was launched at 21st Conference of Parties (COP21) meeting in December, 2015.

The MYC is an initiative combining urban mobility objectives and climate considerations. It aims at
providing solutions in a fully integrated manner, analysing different modes of transportation within
the urban fabric, with the objective of providing people long-term, sustainable, adequate, reliable
and cost-efficient transportation opportunities.

The project seeks to back 100 cities worldwide in three years, which are engaged in sustainable
urban mobility planning to reduce greenhouse gas emissions.

In India, the MYC aims at supporting three pilot cities viz. Nagpur, Kochi and Ahmedabad in their
efforts to reduce their Green House Gas (GHG) emissions related to urban transport by
implementing urban mobility plans at local level and to help India at national level to improve their
sustainable transport policy.

The three pilot cities selected under the programme as well as MoHUA will benefit from the
Technical Assistance activities.

Q Source:
http://www.insightsonindia.com/2018/09/07/insights-daily-current-affairs-07-september-2018/

17 Which of these is/are the key differences between Prorogation and Adjournment of the House?
1. Prorogation only terminates a sitting and not a session of the House, unlike Adjournment.
2. Prorogation lapses all business pending before the House and the same cannot be resumed when
the House meets again, unlike adjournment.

Which of the above is/are correct?


A. 1 only
B. 2 only

13
https://telegram.me/UPSCMaterials https://telegram.me/FreeUPSCMaterials https://telegram.me/MaterialforExam
Mock Test 11

C. Both 1 and 2
D. None

Correct Answer : D

Answer Justification :

Justification: Adjournment only terminates a sitting and not a session of the House. Prorogation
not only terminates a sitting but also a session of the House.

Adjournment is done by presiding officer of the House; whereas Prorogation is done by the
president of India.

Adjournment does not affect the bills or any other business pending before the House;
whereas Prorogation also does not affect the bills or any other business pending before the
House.

However, all pending notices (other than those for introducing bills) lapse on prorogation and
fresh notices have to be given for the next session.

Q Source: Chapter – Parliament: Indian Polity: M Laxmikanth

18 A starred question
1. Requires a written answer
2. Supplementary questions cannot follow
3. Must be asked by giving a notice of less than ten days

Select the correct answer using the codes below.


A. 1 and 2 only
B. 2 and 3 only
C. 1 only
D. None of the above

Correct Answer : D

Answer Justification :

Justification: The questions are of three kinds, namely, starred, unstarred and short notice.

A starred question (distinguished by an asterisk) requires an oral answer and hence supplementary
questions can follow.

An unstarred question, on the other hand, requires a written answer and hence, supplementary

14
https://telegram.me/UPSCMaterials https://telegram.me/FreeUPSCMaterials https://telegram.me/MaterialforExam
Mock Test 11

questions cannot follow.

A short notice question is one that is asked by giving a notice of less than ten days. It is answered
orally.

Q Source: Chapter – Parliament: Indian Polity: M Laxmikanth

19 The speculative demand for money is based on which of the following observations?
1. When the interest rate is very high everyone expects it to fall in future and thus convert their money
into bonds.
2. All the people have the same expectations regarding the future movements in the market rate of
interest.

Which of the above is/are correct?


A. 1 only
B. 2 only
C. Both 1 and 2
D. None

Correct Answer : A

Answer Justification :

Justification: The price of a bond is inversely related to the market rate of interest.

Different people have different expectations regarding the future movements in the market rate of
interest based on their private information regarding the economy. If you think that the market rate
of interest should eventually settle down to 8 per cent per annum, then you may consider the
current rate of 5 per cent too low to be sustainable over time.

You expect interest rate to rise and consequently bond prices to fall. If you are a bond holder
a decrease in bond price means a loss to you – similar to a loss you would suffer if the value of
a property held by you suddenly depreciates in the market. Such a loss occurring from a
falling bond price is called a capital loss to the bond holder.

Under such circumstances, you will try to sell your bond and hold money instead. Thus
speculations regarding future movements in interest rate and bond prices give rise to the
speculative demand for money.

When the interest rate is very high everyone expects it to fall in future and hence anticipates
capital gains from bond-holding. Hence people convert their money into bonds. Thus,
speculative demand for money is low.

When interest rate comes down, more and more people expect it to rise in the future and

15
https://telegram.me/UPSCMaterials https://telegram.me/FreeUPSCMaterials https://telegram.me/MaterialforExam
Mock Test 11

anticipate capital loss. Thus they convert their bonds into money giving rise to a high
speculative demand for money. Hence speculative demand for money is inversely related to
the rate of interest.

th
Q Source: Page 37: Chapter 3: 12 NCERT: Macroeconomics

20 A Point of Order is raised in the Parliament when


1. A member of the house has made a factual error in his/her speech
2. The privilege of a member of the Parliament or the house is breached

Which of the above is/are correct?


A. 1 only
B. 2 only
C. Both 1 and 2
D. None

Correct Answer : D

Answer Justification :

Justification: A member can raise a point of order when the proceedings of the House do not
follow the normal rules of procedure.

A point of order should relate to the interpretation or enforcement of the Rules of the House or such
articles of the Constitution that regulate the business of the House and should raise a question that
is within the cognizance of the Speaker.

It is usually raised by an opposition member in order to control the government. It is an


extraordinary device as it suspends the proceedings before the House. No debate is allowed on a
point of order

Q Source: Chapter – Parliament: Indian Polity: M Laxmikanth

21 The value of the currency notes and coins is derived from the guarantee
1. Based on their intrinsic value
2. Provided by the issuing authority of these items
3. That they cannot be refused by anyone as a mode of payment

Select the correct answer using the codes below.


A. 1 and 2 only
B. 1, 2 and 3
C. 2 and 3 only
D. 1 and 3 only

16
https://telegram.me/UPSCMaterials https://telegram.me/FreeUPSCMaterials https://telegram.me/MaterialforExam
Mock Test 11

Correct Answer : C

Answer Justification :

Justification: Every currency note bears on its face a promise from the Governor of RBI that if
someone produces the note to RBI, or any other commercial bank, RBI will be responsible for giving
the person purchasing power equal to the value printed on the note.

The same is also true of coins. Currency notes and coins are therefore called fiat money. They do
not have intrinsic value like a gold or silver coin. They are also called legal tenders as they cannot
be refused by any citizen of the country for settlement of any kind of transaction.

Cheques drawn on savings or current accounts, however, can be refused by anyone as a mode of
payment. Hence, demand deposits are not legal tenders.
th
Q Source: Page 37: Chapter 3: 12 NCERT: Macroeconomics

22 The difference between narrow and broad money is based on their


1. Acceptability in general public
2. Legitimacy in international transactions
3. Liquidity

Select the correct answer using the codes below.


A. 1 only
B. 1 and 3 only
C. 3 only
D. 2 and 3 only

Correct Answer : C

Answer Justification :

Justification: M0 and M1, also called narrow money, normally include coins and notes in
circulation and other money equivalents that are easily convertible into cash. M2 includes M1 plus
short-term time deposits in banks and 24-hour money market funds.

M3 includes M2 plus longer-term time deposits and money market funds with more than 24-hour
maturity. The exact definitions of the three measures depend on the country.

M4 includes M3 plus other deposits. The term broad money is used to describe M2, M3 or M4,
depending on the local practice.
th
Q Source: Page 38: Chapter 3: 12 NCERT: Macroeconomics

23 The expenditure ‘made’ from the Consolidated Fund of India CANNOT be


1. Discussed and voted by the Parliament
2. Considered as a part of the appropriation bill
17
https://telegram.me/UPSCMaterials https://telegram.me/FreeUPSCMaterials https://telegram.me/MaterialforExam
Mock Test 11

Which of the above is/are correct?


A. 1 only
B. 2 only
C. Both 1 and 2
D. None

Correct Answer : D

Answer Justification :

Justification: The budget consists of two types of expenditure—the expenditure ‘charged’ upon the
Consolidated Fund of India and the expenditure ‘made’ from the Consolidated Fund of India.

The charged expenditure is non-votable by the Parliament, that is, it can only be discussed by the
Parliament, while the other type has to be voted by the Parliament.

The latter forms part of the appropriation bill, and voted by the Parliament, for e.g. defence
expenditure.

Q Source: Chapter – Parliament: Indian Polity: M Laxmikanth

24 The UNESCO has introduced the designation ‘Biosphere Reserve’ for natural areas to minimize
conflict between

A. Conservation of exotic v/s non-exotic species


B. Development and Conservation
C. Backwash and Spread effects
D. Terrestrial and non-terrestrial ecosystems

Correct Answer : B

Answer Justification :

Justification: Biosphere Reserves (BRs) are representative parts of natural and cultural landscapes
extending over large area of terrestrial or coastal/marine ecosystems or a combination thereof and
representative examples of bio-geographic zones/provinces.

Option D: Since they involve both systems, D is wrong

Option A: Exotic species may hurt the local ecosystem, and that’s not the point behind establishing
a biosphere reserve. A BR preserves the natural landscape as it is. So, A is wrong.

Option C: Backwash effects refer to the deleterious effect of development for a backward region.
For e.g. saving deposits from rural areas are transferred to urban areas for investment projects,
without rural areas receiving much in return. This is purely a balanced development related
concept, and doesn’t link much to ecological conservation. So, C is an inappropriate answer.

18
https://telegram.me/UPSCMaterials https://telegram.me/FreeUPSCMaterials https://telegram.me/MaterialforExam
Mock Test 11

Areas potential for preservation of traditional tribal or rural modes of living for harmonious use of
environment are also covered in Biosphere reserves.

Q Source: Revision: Improvised UPSC Qs

25 “It shall be the endeavour of every state and every local authority within the state to provide
adequate facilities for instruction in the mother-tongue at the primary stage of education to children
belonging to linguistic minority groups.” This is a
1. Provision made under an Act of Parliament
2. Justiciable in courts if not implemented by the Government of India
3. Forms part of Fundamental rights under Part III of the Constitution

Select the correct answer using the codes below.


A. 1 only
B. 2 and 3 only
C. 1 and 3 only
D. None of the above

Correct Answer : D

Answer Justification :

Justification: It forms a part of Directive principles.

Apart from the Directives included in Part IV, there are some other Directives contained in other
Parts of the Constitution. Instruction in mother tongue is a part of Article 350-A in Part XVII.

Hence, all the above statements 1, 2 and 3 are wrong, as directive principles are non-justiciable.

Learning: Some other such directives are:

Claims of SCs and STs to Services: The claims of the members of the Scheduled Castes and
the Scheduled Tribes shall be taken into consideration, consistently with the maintenance of
efficiency of administration, in the making of appointments to services and posts in
connection with the affairs of the Union or a State (Article 335 in Part XVI).

Development of the Hindi Language: It shall be the duty of the Union to promote the spread of
the Hindi language and to develop it so that it may serve as a medium of expression for all the
elements of the composite culture of India (Article 351 in Part XVII).

Q Source: Revision: Indian Polity: M Laxmikanth

26 The role of the monetary authority is known as the lender of last resort because it prevents

19
https://telegram.me/UPSCMaterials https://telegram.me/FreeUPSCMaterials https://telegram.me/MaterialforExam
Mock Test 11

A. Unsustainable budget deficits


B. Situations of bank run
C. Balance of Payments crisis
D. All of the above

Correct Answer : B

Answer Justification :

Justification: Every informed individual in the economy keeps his money in a bank account. Why
do they still keep their money in bank deposits when they are aware of the possibility of default by
their banks in case of a bank run (a situation where everybody wants to take money out of one’s
bank account before the bank runs?

The Reserve Bank of India plays a crucial role here. In case of a crisis like the above it stands by the
commercial banks as a guarantor and extends loans to ensure the solvency of the latter.

This system of guarantee assures individual account-holders that their banks will be able to pay
their money back in case of a crisis and there is no need to panic thus avoiding bank runs. This role
of the monetary authority is known as the lender of last resort.

Q Source: Page 43: Chapter 3: 12th NCERT: Macroeconomics

27 Which among the following can authorize the Parliament to create new All-India Services common to
both the Centre and states?

A. President of India
B. Speaker, Lok Sabha
C. Rajya Sabha
D. Inter-State Council

Correct Answer : C

Answer Justification :

Learning: Due to its federal character, the Rajya Sabha has been given two exclusive or special
powers that are not enjoyed by the Lok Sabha:

1. It can authorise the Parliament to make a law on a subject enumerated in the State List (Article
249).

2. It can authorise the Parliament to create new All-India Services common to both the Centre and
states (Article 312).

An analysis of the above points makes it clear that the position of the Rajya Sabha in our
constitutional system is not as weak as that of the House of Lords in the British constitutional

20
https://telegram.me/UPSCMaterials https://telegram.me/FreeUPSCMaterials https://telegram.me/MaterialforExam
Mock Test 11

system nor as strong as that of the Senate in the American constitutional system.

Except in financial matters and control over the council of ministers, the powers and status of the
Rajya Sabha in all other spheres are broadly equal and coordinate with that of the Lok Sabha.

Q Source: Chapter – Parliament: Indian Polity: M Laxmikanth

28 Open Market Operations (OMOs) by the Central Bank invariably lead to


1. Lower money supply
2. Reduced foreign exchange
3. Higher statutory ratios

Select the correct answer using the codes below.


A. 1 and 2 only
B. 3 only
C. 2 and 3 only
D. None of the above

Correct Answer : D

Answer Justification :

Justification: Statement 1: RBI purchases (or sells) government securities to the general public in
a bid to increase (or decrease) the stock of high powered money in the economy. This is called
OMOs. So, 1 is wrong as it is not always the case.

Statement 2: Only when OMOs are used for sterilization, do we see a change in the forex reserves.
If used only in the domestic market, forex does not change. So, 2 is wrong as well.

Statement 3: These remain unaffected by OMOs and decided independently by the Central bank (or
MPC now).

Q Source: Page 43: Chapter 3: 12th NCERT: Macroeconomics

29 The Constitution does not contain any specific procedure for the selection and appointment of the
Prime Minister. This means that
1. The Constitution does not mention the appointing authority for the office of Prime Minister.
2. The President is free to change the selection and appointment procedure of Prime Minister by rules
issued in the Gazette of India.

Which of the above is/are correct?


A. 1 only
B. 2 only
C. Both 1 and 2
D. None

21
https://telegram.me/UPSCMaterials https://telegram.me/FreeUPSCMaterials https://telegram.me/MaterialforExam
Mock Test 11

Correct Answer : D

Answer Justification :

Justification: Statement 1: Article 75 says only that the Prime Minister shall be appointed by the
president. However, this does not imply that the president is free to appoint any one as the Prime
Minister.

Statement 2: In accordance with the conventions of the parliamentary system of government, the
President has to appoint the leader of the majority party in the Lok Sabha as the Prime Minister.

But, when no party has a clear majority in the Lok Sabha, then the President may exercise his
personal discretion in the selection and appointment of the Prime Minister. In such a situation, the
President usually appoints the leader of the largest party or coalition in the Lok Sabha as the Prime
Minister and asks him to seek a vote of confidence in the House within a month.

However, this is not guided by rules made by the President, and is based on established
conventions. So, 2 is wrong.

Q Source: Chapter - Prime Minister: Indian Polity: M Laxmikanth

30 The Ministry of Corporate Affairs (MCA) has notified the dematerialisation of securities with effect
from 2nd October, 2018, for the issue of further shares and transfer of all shares by unlisted public
companies. What are the benefits of such a move?
1. Elimination of risks associated with physical certificates such as mutilation and fraud
2. Ease in transfer and pledge of securities.

Which of the above is/are correct?


A. 1 only
B. 2 only
C. Both 1 and 2
D. None

Correct Answer : C

Answer Justification :

Concept: In finance and financial law, dematerialization refers to the substitution of paper-form
securities by book-entry securities. This is a form of indirect holding system where an intermediary,
such as a broker or central securities depository, or the issuer itself (e.g., French system) holds a
record of the ownership of shares usually in electronic format.

The dematerialization of securities such as stocks has been a major trend since the late 1960s, with
the result that by 2010 the majority of global securities were held in dematerialized form.

Justification: The MCA has taken this step as a measure for further enhancing transparency,
investor protection and governance in the corporate sector. The rules in this regard have been
22
https://telegram.me/UPSCMaterials https://telegram.me/FreeUPSCMaterials https://telegram.me/MaterialforExam
Mock Test 11

amended accordingly.

According to the MCA, the major benefits of dematerialisation of securities which will now be
available to unlisted Public companies include:-

Elimination of risks associated with physical certificates such as loss, theft, mutilation, fraud
etc.

Improving the corporate governance system by increasing transparency and preventing mal-
practices such as benami shareholding, back dated issuance of shares, etc.

Exemption from payment of stamp duty on transfer.

Ease in transfer, pledge etc. of securities.

Learning: Accredited social health activists (ASHAs) are community health workers instituted by the
government of India's Ministry of Health and Family Welfare (MoHFW) as a part of the National
Rural Health Mission (NRHM).

Q Source: http://pib.nic.in/PressReleseDetail.aspx?PRID=1545620

31 With reference to money market, consider the following statements.


1. It allows the borrowing and lending of funds in the short-term
2. Institutions operating in the money market are outside of the ambit of regulation of RBI.

Which of the above is/are correct?


A. 1 only
B. 2 only
C. Both 1 and 2
D. None

Correct Answer : A

Answer Justification :

Justification: Statement 1: Money market is concerned with borrowing and lending upto a period
of 3-years, for e.g. treasury bills (365 days), or repo rate based borrowing from banks etc come
within this market.

Capital market engages in long-term lending and borrowing.

Statement 2: Commercial banks, regional rural banks etc. that give short-term funds are also a part
of this market, and are well regulated by RBI.

23
https://telegram.me/UPSCMaterials https://telegram.me/FreeUPSCMaterials https://telegram.me/MaterialforExam
Mock Test 11

Q Source: Page 91: Shankar Ganesh: Indian Economy – Key Concepts

32 Consider the following statements.


1. The Government of India has the powers to print and issue currency notes of Rs. 1 and Rs. 2.
2. The Government of India has the sole right to mint coins of all denominations.
3. Coinage Act of 2011 provides that the central government may authorise minting of coin of
denomination not higher than Rs. 1,000.

Select the correct answer using the codes below.


A. 1, 2 and 3 only
B. 2 only
C. 2 and 3 only
D. None of the above

Correct Answer : C

Answer Justification :

Background: With the repeal of a section of a Currency Ordinance, the RBI was of the view that,
the Government of India is not empowered to issue note of denominational value of one rupee, in
such a case, the law ministry opinion was taken.

Justification: Statement 1, 2 and 3: The Law Ministry in its opinion stated that the Coinage Act of
2011, which consolidates the laws relating to coinage and the mints, does not bar the Government
of India from printing one rupee notes.

Section 4 of the Act provides that the central government may authorise minting of coin of
denomination not higher than Rs. 1,000, it said, adding that the definition of coin in the Act makes it
clear that Government of India one rupee note is included in the definition of coin.

The Act defines a coin as “made of any metal or any other material stamped by the Government or
any other authority empowered by the Government in this behalf and which is a legal tender
including commemorative coin and Government of India one rupee note,” the ministry said in its
opinion.

Central government is not precluded to issue one rupee Government of India note under the
Coinage Act, 2011. The dimension, design, material and standard weight of such One Rupee Note
have to be prescribed by the Central Government in terms of Section 4 and 5 of the Coinage Act,
2011,” the opinion said.

Note: The printing of notes in the denominations of Re. 1 and Rs. 2 has been discontinued as these
denominations have been coinised. However, such notes issued earlier are still in circulation.

Q Source: Additional Research: Page 92: Shankar Ganesh: Indian Economy – Key Concepts

33 High and volatile Inflation generally tends to

24
https://telegram.me/UPSCMaterials https://telegram.me/FreeUPSCMaterials https://telegram.me/MaterialforExam
Mock Test 11

1. Reduce value and demand of national currency


2. Increase lending activities by financial institutions

Which of the above is/are correct?


A. 1 only
B. 2 only
C. Both 1 and 2
D. None

Correct Answer : A

Answer Justification :

Justification: Statement 1: Inflation erodes the value of currency. For e.g. if inflation in India is
1000%, then the very next year the value of Rs. 5000 in year 2016 will be near Rs. 500 in year 2017.
It also leads to depreciation in the exchange rate of the currency.

As a result, people lose confidence in the currency and holding it becomes a risky proposition.

So, people switch to other forms of wealth like Gold, Foreign Currency, i.e. “inflation proof” assets.

Statement 2: If inflation is high and volatile, the returns on lending are uncertain, volatile and low
too. For e.g. if a bank has lent to a customer at an interest rate of 10% p.a., but inflation is hovering
between 4-14%, the bank can make a profit of 6% (real interest rate) or bear a loss of 4% depending
on whether the inflation is high or low. This discourages lending.

Q Source: Chapter 9: Shankar Ganesh: Indian Economy – Key Concepts

34 The Reserve Bank of India acts as a banker to which of the following apart from the Central
government?
1. State governments
2. State of Jammu and Kashmir
3. Local governments

Select the correct answer using the codes below.


A. 1 only
B. 1 and 2 only
C. 1 and 3 only
D. None of the above

Correct Answer : A

Answer Justification :

Justification: The RBI acts as banker to the government the Central as well as state governments.
As such, it transacts all banking business of the government, which involves the receipt and

25
https://telegram.me/UPSCMaterials https://telegram.me/FreeUPSCMaterials https://telegram.me/MaterialforExam
Mock Test 11

payment of money on behalf of the government and carrying out of its exchange, remittance and
other banking operations.

Statement 2 and 3: The state of J&K is not covered under this scheme, and local governments are
not taken care of by the RBI.

Learning: As government’s banker, the RBI provides short-term credit to the government to meet
any shortfalls in its receipts over its disbursements. It also provides short-term credit to state
governments as ways and means advances. But, some state governments do resort to over-drafts at
times for short periods. The RBI has not been able to stop this practice.

As government’s banker, the RBI is also charged with the responsibility of managing the public (i.e.,
the government) debt. In discharge of this responsibility, the RBI manages all new issues of
government loans, services the public debt outstanding, and nurses the market for government
securities.

Q Source: Additional Research: Page 92: Shankar Ganesh: Indian Economy – Key Concepts

35 Consider the following statements regarding the Marginal Standing Facility (MSF).
1. MSF functions as the last resort for banks to borrow short-term funds over and above that available
under the Liquidity Adjustment Facility Window (LAF).
2. MSF is an extraordinary rate at which banks can borrow money from the central bank by a much
cheaper route than repo rate.

Which of the above is/are correct?


A. 1 only
B. 2 only
C. Both 1 and 2
D. None

Correct Answer : A

Answer Justification :

Justification: Marginal Standing Facility (MSF) is a new scheme announced by the Reserve Bank
of India (RBI) in its Monetary Policy (2011-12) and refers to the penal rate at which banks can
borrow money from the central bank over and above what is available to them through the LAF
window.

Statement 2: MSF, being a penal rate, is always fixed above the repo rate. So, 2 is incorrect.

The MSF would be the last resort for banks once they exhaust all borrowing options including
the liquidity adjustment facility by pledging government securities, where the rates are lower
in comparison with the MSF.

Banks can borrow funds by pledging government securities within the limits of the statutory

26
https://telegram.me/UPSCMaterials https://telegram.me/FreeUPSCMaterials https://telegram.me/MaterialforExam
Mock Test 11

liquidity ratio.

The scheme has been introduced by RBI with the main aim of reducing volatility in the
overnight lending rates in the inter-bank market and to enable smooth monetary transmission
in the financial system.

Q Source: Page 96: Chapter 6: Shankar Ganesh: Indian Economy – Key Concepts

36 Consider the following about use of the term “Sarvodaya” in India.


1. It was associated with the translation of John Ruskin's tract on political economy, Unto This Last, by
Gandhi.
2. It was the name for a social movement in post-independence India.
3. It was used as a name to call the tirtha of Mahavira.

Select the correct answer using the codes below.


A. 1 and 2 only
B. 2 and 3 only
C. 3 only
D. 1, 2 and 3

Correct Answer : D

Answer Justification :

Justification: Statement 1: It is a Sanskrit term meaning 'universal uplift' or 'progress of all'. The
term was used by Mahatma Gandhi as the title of his 1908 translation of John Ruskin's tract on
political economy, Unto This Last, and Gandhi came to use the term for the ideal of his own political
philosophy

Statement 2: Later Gandhians, like the Indian nonviolence activist Vinoba Bhave, embraced the
term as a name for the social movement in post-independence India which strove to ensure that
self-determination and equality reached all strata of Indian society.

Statement 3: Samantabhadra, an illustrious Digambara monk, as early as 2nd century A.D., called
the tirtha of Mahavira (24th Tirthankara) by the name sarvodaya.
th
Q Source: Additional Research: Chapter on Emergency: 12 NCERT: Politics in India after
Independence

37 Consider the following statements.


1. Regional Rural Banks (RRBs) have been set up in the country on the sponsorship of individual
nationalised commercial banks.
2. The State Co-operative Banks (SCBs) are allowed to lend money only for non-agricultural purposes,
small borrowers and businesses.

27
https://telegram.me/UPSCMaterials https://telegram.me/FreeUPSCMaterials https://telegram.me/MaterialforExam
Mock Test 11

Which of the above is/are correct?


A. 1 only
B. 2 only
C. Both 1 and 2
D. None

Correct Answer : A

Answer Justification :

Justification: Statement 1: These banks aim at taking the banking facilities to the doorsteps of
rural masses especially in the remote areas.

The objective was to provide credit to small and marginal farmers, agricultural labourers, artisans
and small entrepreneurs so as to develop productive activities in the rural areas.

They have been conceived as institutions that combine the features of both the co-operatives and
commercial banks. Initially, five RRBs were set up in 1975, at Moradabad and Gorkhpur in Uttar
Pradesh; Bhiwani in Haryana; Jaipur in Rajasthan and Malda in West Bengal.

But gradually the spread of these banks has increased and the Government has taken several policy
measures for their growth and expansion.

Statement 2: The State Co-operative Banks (SCBs) constitute the apex of the three tier co-operative
credit structure, organised at the level of individual States. While, Urban Co-operative Banks
(UCBs), refers to the primary cooperative banks located in urban and semi-urban areas.

Initially, these banks were allowed to lend money only for non-agricultural purposes and
essentially to small borrowers and businesses. Today, their scope of operations has widened
considerably. The Urban Banks Department of the Reserve Bank of India is vested with the
responsibility of regulating and supervising the urban cooperative banks.

Please read this comprehensive article on developments in co-operative banking in India.

https://www.rbi.org.in/scripts/PublicationsView.aspx?id=18062

Q Source: Additional Research: Page 96: Shankar Ganesh: Indian Economy – Key Concepts

38 The International Date Line (IDL) is an imaginary line of longitude on the Earth’s surface located at
about 180 degrees east (or west) of the Greenwich Meridian. It passes through
1. Australia
2. New Zealand
3. Japan
4. Mongolia

Select the correct answer using the codes below.


A. 1 and 2 only
B. 3 and 4 only

28
https://telegram.me/UPSCMaterials https://telegram.me/FreeUPSCMaterials https://telegram.me/MaterialforExam
Mock Test 11

C. 2 only
D. None of the above

Correct Answer : D

Answer Justification :

Learning: The line divides the Earth’s eastern and western hemispheres just as the equator divides
the northern and southern hemispheres.

If you travel around the world, changing standard time by one hour each time you enter a new time
zone, then a complete circuit would mean that you adjusted your clock or watch time by 24 hours.

This would lead to a difference of one day between the date on your clock and the real calendar
date.

To avoid this, countries are on either side of the International Date Line which runs down the
middle of the Pacific Ocean. If you cross the date line moving east, you subtract a day, whereas if
you are moving west you add a day.

Q Source: Revision: Previous test syllabus

39 Which of the following has been chosen as the first National Implementing Entity (NIE) of Green
Climate Fund (GCF) established under United Nations Framework Convention on Climate Change
(UNFCCC)?

A. National Bank for Agriculture and Rural Development (NABARD)

29
https://telegram.me/UPSCMaterials https://telegram.me/FreeUPSCMaterials https://telegram.me/MaterialforExam
Mock Test 11

B. National Green Corps


C. NITI Aayog
D. Centre for Science and Environment, India

Correct Answer : A

Answer Justification :

Learning: The decision to accredit NABARD was taken in 2015 at the Board meeting of GCF.

The NIEs are those national legal entities that are recognized by the Board as meeting the
fiduciary standards established by the Board.

The NIEs bear full responsibility for the overall management of the projects and programmes
financed by the Green Climate Fund and have all financial, monitoring, and reporting
responsibilities.

NABARD has earlier been accredited by the Adaptation Fund Board (AFB) of UNFCCC as the
only National Implementing Entity (NIE) for India.

Q Source: Additional Research: NABARD in Chapter 6 Shankar Ganesh: Indian Economy – Key
Concepts

40 With reference to the Lead Bank scheme, consider the following statements.
1. The scheme envisages assignment of lead roles to individual banks, both in public sector and
private sector, for the districts allotted to them.
2. The lead bank acts as a leader for coordinating the efforts of all credit institutions in the allotted
districts to increase the flow of credit to agriculture and small scale industries.

Which of the above is/are correct?


A. 1 only
B. 2 only
C. Both 1 and 2
D. None

Correct Answer : C

Answer Justification :

Justification: The Lead Bank Scheme, introduced towards the end of 1969, envisages assignment
of lead roles to individual banks (both in public sector and private sector) for the districts allotted to
them.

30
https://telegram.me/UPSCMaterials https://telegram.me/FreeUPSCMaterials https://telegram.me/MaterialforExam
Mock Test 11

A bank having a relatively large network of branches in the rural areas of a given district and
endowed with adequate financial and manpower resources has generally been entrusted with the
lead responsibility for that district.

Accordingly, all the districts in the country have been allotted to various banks. The lead bank acts
as a leader for coordinating the efforts of all credit institutions in the allotted districts to increase
the flow of credit to agriculture, small-scale industries and other economic activities included in the
priority sector in the rural and semi-urban areas, with the district being the basic unit in terms of
geographical area.

Q Source: Page 108: Shankar Ganesh: Indian Economy – Key Concepts

41 In which of these cases the Rajya Sabha has an equal Status with Lok Sabha?
1. Amending or rejecting a Money Bill
2. Voting on the demands for grants
3. Discussing and criticising the policies of the government
4. Ascertaining the confidence of the government on the house floor

Select the correct answer using the codes below.


A. 2 and 3 only
B. 1, 3 and 4 only
C. 3 only
D. 1 and 4 only

Correct Answer : C

Answer Justification :

Justification: Statement 1: A Money Bill can be introduced only in the Lok Sabha and not in the
Rajya Sabha.

Rajya Sabha cannot amend or reject a Money Bill. It should return the bill to the Lok Sabha within
14 days, either with recommendations or without recommendations.

Statement 2: Rajya Sabha can only discuss the budget but cannot vote on the demands for grants
(which is the exclusive privilege of the Lok Sabha).

Statement 3 and 4: Moreover, the Rajya Sabha cannot remove the council of ministers by passing a
no-confidence motion.

This is because the Council of ministers is collectively responsible only to the Lok Sabha.

But, the Rajya Sabha can discuss and criticise the policies and activities of the government.

Q Source: Chapter – Parliament: Indian Polity: M Laxmikanth

31
https://telegram.me/UPSCMaterials https://telegram.me/FreeUPSCMaterials https://telegram.me/MaterialforExam
Mock Test 11

42 Consider the following statements about types of trading.


1. The execution of a futures agreement by the buyer and seller is compulsory and not optional.
2. An ‘options’ agreement is a buying and selling of securities on the same day with an option for the
buyer not to enforce the agreement.

Which of the above is/are correct?


A. 1 only
B. 2 only
C. Both 1 and 2
D. None

Correct Answer : A

Answer Justification :

Justification: These are the major types of trading.

In Cash trading, the sale and purchase of securities takes place in the prevailing price on the day of
trading. In Forward trading, both buyer and seller agree to buy and sell respectively at a future
date at a pre agreed price, irrespective of the price that prevails on the day of trade.

There are two types of forward trade. One is Futures and another one is Options. Though both are
forward trade methods, there is a slight difference. In case of Futures, both buyer and seller have to
execute the agreement.

In case of Options, the buyer or seller can withdraw from the agreement. To have this option the
buyer or seller has to deposit some amount as premium.

In the case when he fails to execute the agreement, he has to forego the premium amount. The
choice available to seller not to execute the agreement is called Put option. If it is available to buyer
it is called Call option.

Q Source: Chapter 7: Shankar Ganesh: Indian Economy – Key Concepts

43 Article 123 of the Constitution empowers the President to promulgate ordinances. Consider the
following in this regard.
1. An ordinance made when both the Houses are in session is valid if approved by the Council of
Ministers and Parliament.
2. An ordinance cannot be used to amend the constitution.

Which of the above is/are correct?


A. 1 only
B. 2 only
C. Both 1 and 2
D. None

32
https://telegram.me/UPSCMaterials https://telegram.me/FreeUPSCMaterials https://telegram.me/MaterialforExam
Mock Test 11

Correct Answer : B

Answer Justification :

Justification: Statement 1: He can promulgate an ordinance only when both the Houses of
Parliament are not in session or when either of the two Houses of Parliament is not in session.

An ordinance can also be issued when only one House is in session because a law can be
passed by both the Houses and not by one House alone.

An ordinance made when both the Houses are in session is void. Thus, the power of the
President to legislate by ordinance is not a parallel power of legislation.

Statement 2: This is because a constitutional amendment requires special majority in both houses of
Parliament, unlike an ordinary legislation that can be approved by a simple majority.

Moreover, some amendment may require the approval of half of the states as well. Hence, 2 is
correct.

Q Source: Chapter 17: Indian Polity: M Laxmikanth

44 The Program for International Student Assessment (PISA) is an international assessment that
measures

A. Fifteen-year-old students’ reading, mathematics, and science literacy every three years.
B. The communication skills of high school and undergraduate students from the best
universities across the world
C. The competence of industrialized countries in preparing leaders for tomorrow
D. The relative performance in Maths and Science Olympiads of select nations every five years

Correct Answer : A

Answer Justification :

Learning: First conducted in 2000, the major domain of study rotates between reading,
mathematics, and science in each cycle.

PISA also includes measures of general or cross-curricular competencies, such as collaborative


problem solving.

By design, PISA emphasizes functional skills that students have acquired as they near the end of
compulsory schooling.

PISA is coordinated by the Organization for Economic Cooperation and Development (OECD), an
intergovernmental organization of industrialized countries, and is conducted in the United States by

33
https://telegram.me/UPSCMaterials https://telegram.me/FreeUPSCMaterials https://telegram.me/MaterialforExam
Mock Test 11

NCES. Data collection for the most recent assessment was completed in Fall 2015.

In 2012 PISA test, schools of Shanghai in China topped reading, mathematics and science test,
followed closely by Singapore. In 2015, Singapore, Japan and Estonia were ranked as top three
countries, in that order.

Union Ministry of Human Resource Development (HRD has officially decided to participate in
Programme for International Student Assessment (PISA) after gap of nine years. It will send team of
officers to Paris to negotiate India’s terms of participation in PISA 2021.

India had taken part in Programme for International Student Assessment (PISA) in 2009 and
bagged the 72nd rank among 74 participating countries. Then UPA government had boycotted
PISA, blaming “out of context” questions for India’s dismal performance.

Q Source: AS mentioned above

45 Which of the following factors will give the strongest upwards push to inflation?

A. An increase in the salary of government employees financed from domestic borrowing


B. An increase in the salary of government employees financed from printing new currency
C. An increase in the salary of government employees financed from revenue resources without
causing a fiscal deficit
D. An increase in the salary of government employees financed from cutting government
expenditure in other sectors

Correct Answer : B

Answer Justification :

Justification: An increase in salary pumps more money in the economy which creates more
demand for goods and services, and leads to inflation in the short-term.

However, this money supply will NOT increase if resources were shifted from one part of the
economy (lets say maintaining roads) to the other (increasing salary of employees). This is the case
in Option C and Option D.

In option A, there is a high chance of inflation because money is being borrowed (which may have
lied dormant) to finance this salary increase. But, the chance is the highest in option B because
there is a fresh stock of money coming in which was non-existent before. In option A, the existing
money was being recycled from one pocket to another pocket, which is not the case with option B.

Q Source: Chapter 9: Shankar Ganesh: Indian Economy – Key Concepts

46 Which of the following is/are common to the oath taken by the Prime Minister and the other Members
of Parliament (MPs)?
1. To bear true faith and allegiance to the Constitution of India
2. To uphold the sovereignty and integrity of India

34
https://telegram.me/UPSCMaterials https://telegram.me/FreeUPSCMaterials https://telegram.me/MaterialforExam
Mock Test 11

3. To do right to all manner of people in accordance with the Constitution and the law, without fear or
favour, affection or ill will.

Select the correct answer using the codes below.


A. 1 and 2 only
B. 2 only
C. 2 and 3 only
D. 3 only

Correct Answer : A

Answer Justification :

Justification: Before the Prime Minister enters upon his office, the president administers to him
the oaths of office and secrecy. In his oath of office, the Prime Minister swears:

To bear true faith and allegiance to the Constitution of India,

to uphold the sovereignty and integrity of India,

to faithfully and conscientiously discharge the duties of his office, and

to do right to all manner of people in accordance with the Constitution and the law, without
fear or favour, affection or ill will.

The MP also takes a similar oath apart from the oath of secrecy and the last point in the oath of PM
(or Union Ministers).

Q Source: Chapter - Prime Minister: Indian Polity: M Laxmikanth

47 Inflation would usually redistribute wealth from


1. Consumers to producers
2. Debtors to creditor

Which of the above is/are correct?


A. 1 only
B. 2 only
C. Both 1 and 2
D. None

Correct Answer : A

35
https://telegram.me/UPSCMaterials https://telegram.me/FreeUPSCMaterials https://telegram.me/MaterialforExam
Mock Test 11

Answer Justification :

Justification: Statement 1: Inflation has impact on all the economic units. It has favourable impact
on some and unfavourable impact on others.

In inflationary situation, the producers stand to gain and consumers stand to lose.

The producer's profit will increase as a result of inflation. The purchasing power of money held by
consumer falls. So, they have to pay more money to purchase the same amount of goods and
services what they bought before inflation. Here, the income of consumer gets transferred from
consumers to producers.

Statement 2: In case of debtor and creditor, debtor is gainer and creditor is loser. Take an example.
The debtor borrowed for interest at the rate of 5 % a day and debtor is a mango vendor. He has to
repay Rs 105 on the next day. The price of mango on day one is Rs. 10 per mango. The debtor can
buy 10 mangoes.

On day two, the price of mango is Rs. 15. The debtor can sell 10 mangoes for Rs. 150. The debtor
can repay his debt by selling only 7 mangoes. So he gains Rs. 45 or 3 mangoes. The creditor can
buy only 7 mangoes with Rs. 105 he got back. Suppose, he purchased mango on day one instead of
lending, he may have bought 10 mangoes. So he loses 3 mangoes. This relation holds true for
private as well as public debt.

Q Source: Chapter 9: Shankar Ganesh: Indian Economy – Key Concepts

48 What is the difference in the electoral college of the Vice-President and the President of India?
1. Both the houses take part in the election of President, unlike the Vice-President who is elected by
the Rajya Sabha alone.
2. State legislative assemblies do not take part in the election of the Vice-President, unlike that of the
President.

Which of the above is/are correct?


A. 1 only
B. 2 only
C. Both 1 and 2
D. None

Correct Answer : B

Answer Justification :

Justification: Statement 1: The Vice-President, like the president, is elected not directly by the
people but by the method of indirect election. He is elected by the members of an electoral college
consisting of the members of both Houses of Parliament.

Statement 2: His electoral college is different from the Electoral College for the election of the
President in the following two respects:

36
https://telegram.me/UPSCMaterials https://telegram.me/FreeUPSCMaterials https://telegram.me/MaterialforExam
Mock Test 11

It consists of both elected and nominated members of the Parliament (in the case of president,
only elected members).

It does not include the members of the state legislative assemblies (in the case of President,
the elected members of the state legislative assemblies are included).

Q Source: Chapter 17 and 18: Indian Polity: M Laxmikanth

49 Consider the following with reference to the Motion of Thanks.


1. It is addressed by the Leader of the House.
2. The motion is put to vote only in Lok Sabha.
3. It is addressed at the beginning of every new session of the Parliament.

Select the correct answer using the codes below.


A. 1 and 2 only
B. 3 only
C. 1, 2 and 3
D. None of the above

Correct Answer : D

Answer Justification :

Justification: The first session after each general election and the first session of every fiscal year
is addressed by the president.

In this address, the president outlines the policies and programmes of the government in the
preceding year and ensuing year.

This address of the president is called the ‘Motion of Thanks’. At the end of the discussion, the
motion is put to vote.

This motion must be passed in the House. Otherwise, it amounts to the defeat of the government.

This inaugural speech of the president is an occasion available to the members of Parliament to
raise discussions and debates to examine and criticise the government and administration for its
lapses and failures.

Q Source: Chapter – Parliament: Indian Polity: M Laxmikanth

50 As per the Constitution, the Union executive does NOT include

A. President of India
B. Vice President of India
C. Council of Ministers

37
https://telegram.me/UPSCMaterials https://telegram.me/FreeUPSCMaterials https://telegram.me/MaterialforExam
Mock Test 11

D. Advocate general of India

Correct Answer : D

Answer Justification :

Justification: It should be Attorney General of India who is a part of Union executive. Advocate
General is a part of the Stat executive. So, answer is D.

Articles 52 to 78 in Part V of the Constitution deal with the Union executive. The Union executive
consists of the President, the Vice-President, the Prime Minister, the council of ministers and the
attorney general of India.

The President is the head of the Indian State. He is the first citizen of India and acts as the symbol
of unity, integrity and solidarity of the nation.

Q Source: Chapter 17: Indian Polity: M Laxmikanth

51 The famous book “The General Theory of Employment, Interest and Money (1936)” which greatly
shaped future economic thinking was written by

A. Alan Greenspan
B. Adam Smith
C. Robert Lucas
D. John Maynard Keynes

Correct Answer : D

Answer Justification :

Learning: This book is largely credited with creating the terminology and shape of modern
macroeconomics. It sought to bring about a revolution, commonly referred to as the "Keynesian
Revolution”.

The book challenged the established classical economics and showed the importance of state
intervention in the economy.

Modern macroeconomics uses many of its tools for analysing shortfall in demand, inflation,
employment and fluctuations in output.

Q Source: Revision: Previous test syllabus: 12th NCERT: macroeconomics

52 Consider the following statements. Inflation Indexed Bond (IIB)


1. is a bond issued by Non Banking Financial entities
2. gives a constant minimum real return irrespective of inflation level in the economy

38
https://telegram.me/UPSCMaterials https://telegram.me/FreeUPSCMaterials https://telegram.me/MaterialforExam
Mock Test 11

Which of the above is/are correct?


A. 1 only
B. 2 only
C. Both 1 and 2
D. None

Correct Answer : D

Answer Justification :

Justification: Statement 1: It is issued by the Sovereign, which provides the investor a constant
return irrespective of the level of inflation in the economy. The main objective of Inflation Indexed
Bonds is to provide a hedge and to safeguard the investor against macroeconomic risks in an
economy.

Issue of IIBs has assumed significance in the context of high level of inflation experienced in the
Emerging Market and Developing Economies during the recent years, as the value of money loses
rapidly in an environment of high inflation. The issue of Inflation Indexed bonds in advanced
economies is limited on account of low inflation experienced in these economies.

While fixed deposit offers a fixed rate of interest for the investment for a given number of years, it
does not protect the investor from the erosion of real value of the deposit due to inflation. IIB on the
other hand, gives a constant minimum real return irrespective of inflation level in the economy.

Capital increases with the inflation, so actual interest is better than originally promised. In case of
deflation, interest payments decrease with the negative inflation. However, capital does not decline
below the face value, i.e. Initial investment, in case of deflation. The working of IIB is given through
the following example:

Q Source: Additional Research: Chapter 9: Shankar Ganesh: Indian Economy – Key Concepts

53 Before entering upon his office, the President has to make and subscribe to an oath or affirmation.
This oath is administered to the President by

A. Prime Minister of India


B. The predecessor President of India who immediately vacated office

39
https://telegram.me/UPSCMaterials https://telegram.me/FreeUPSCMaterials https://telegram.me/MaterialforExam
Mock Test 11

C. Chief Justice of India


D. Vice-President of India

Correct Answer : C

Answer Justification :

Learning: In his oath, the President swears:

to faithfully execute the office;

to preserve, protect and defend the Constitution and the law; and

to devote himself to the service and well-being of the people of India.

The oath of office to the President is administered by the Chief Justice of India and in his absence,
the seniormost judge of the Supreme Court available.

Any other person acting as President or discharging the functions of the President also undertakes
the similar oath or affirmation.

Q Source: Chapter 17: Indian Polity: M Laxmikanth

54 In India, the geographical distribution of Snow Leopard encompasses


1. Jammu and Kashmir
2. Uttarakhand
3. Arunachal Pradesh
4. Assam

Select the correct answer using the codes below.


A. 1 only
B. 1, 2 and 3 only
C. 2 and 4 only
D. 1 and 2 only

Correct Answer : B

Answer Justification :

Justification: The snow leopard is the state animal of Uttarakhand and the National Heritage
Animal of Pakistan.

Their habitat extends through twelve countries: Afghanistan, Bhutan, China, India, Kazakhstan,

40
https://telegram.me/UPSCMaterials https://telegram.me/FreeUPSCMaterials https://telegram.me/MaterialforExam
Mock Test 11

Kyrgyzstan, Mongolia, Nepal, Pakistan, Russia, Tajikistan, and Uzbekistan. China contains as much
as 60% of all snow leopard habitat areas.

They inhabit alpine and subalpine zones at elevations from 3,000 to 4,500 m (9,800 to 14,800 ft). In
the northern range countries, they also occur at lower elevations.

Officials from Himachal State Wildlife Department have spotted snow leopard in Lippa-Asra wildlife
sanctuary in Kinnaur district of Himachal Pradesh. This finding indicates that snow leopards are
inhabiting new areas.

Learning: In India, their geographical range encompasses a large part of the western Himalayas
including the states of Jammu and Kashmir, Himachal Pradesh, Uttarakhand and Sikkim and
Arunachal Pradesh in the eastern Himalayas.

The snow leopard, like all big cats, is listed on Appendix I of the Convention on International Trade
of Endangered Species (CITES), which makes trading of animal body parts (i.e., fur, bones and
meat) illegal in signatory countries. It is also protected by several national laws in its range
countries.

They are listed as Vulnerable on the IUCN Red List of Threatened Species.

Global Snow Leopard Forum, 2013: 12 countries encompassing the snow leopard’s range
(Afghanistan, Bhutan, China, India, Kazakhstan, Kyrgyz Republic, Mongolia, Nepal, Pakistan,
Russia, Tajikistan, and Uzbekistan).

Bishkek Declaration: To protect the species and it’s environment.

Global Snow Leopard and Eco-system Protection Program: It is a joint initiative of range country
governments, international agencies, civil society, and the private sector. Goal — secure the long-
term survival of the snow leopard in its natural ecosystem.

Q Source:
http://www.insightsonindia.com/2018/09/10/insights-daily-current-affairs-10-september-2018/

55 All executive actions of the Government of India are formally taken in his name. Consider the
following with this reference.
1. He can make rules specifying the manner in which the orders and other instruments made and
executed in his name shall be authenticated.
2. He can make rules for more convenient transaction of business of the Union government, and for
allocation of the said business among the ministers.
3. He can seek any information relating to the administration of affairs of the Union, and proposals for
legislation from the prime minister.

Which of the above is/are correct?


A. 1 and 2 only
B. 2 and 3 only
C. 1 and 3 only
D. 1, 2 and 3

41
https://telegram.me/UPSCMaterials https://telegram.me/FreeUPSCMaterials https://telegram.me/MaterialforExam
Mock Test 11

Correct Answer : D

Answer Justification :

Justification: These are some of the executive powers and functions of the President.

Statement 2: Orders and other instruments made and executed in the name of the President shall
be authenticated in such manner as may be specified in rules to be made by the President.

Some of the other powers are:

He appoints the prime minister and the other ministers. They hold office during his pleasure.

He appoints the attorney general of India and determines his remuneration. The attorney
general holds office during the pleasure of the President.

He appoints the comptroller and auditor general of India, the chief election commissioner and
other election commissioners, the chairman and members of the Union Public Service
Commission, the governors of states, the chairman and members of finance commission, and
so on.

He can seek any information relating to the administration of affairs of the Union, and
proposals for legislation from the prime minister.

He can require the Prime Minister to submit, for consideration of the council of ministers, any
matter on which a decision has been taken by a minister but, which has not been considered
by the council.

Other important function can be found at the Q Source.

Q Source: Chapter 17: Indian Polity: M Laxmikanth

56 Consider the following statements. In India,


1. Venture capital fund is relied upon by several start-ups as ‘seed funding’.
2. Merchant banking services are largely carried away by Non-Banking Financial Companies (NBFCs).

Which of the above is/are correct?


A. 1 only
B. 2 only
C. Both 1 and 2
D. None

42
https://telegram.me/UPSCMaterials https://telegram.me/FreeUPSCMaterials https://telegram.me/MaterialforExam
Mock Test 11

Correct Answer : A

Answer Justification :

Justification: Statement 1: Venture Capital Companies: Usually the financial institutions are
hesitant to finance new products because the profitability Of new products is uncertain and involve
risks. So, to finance such products, separate type of financial companies who venture to finance
them are established. They are called Venture capital companies. The venture capital companies
provide capital to companies that produce new products based on innovations and to new
industries.

Statement 2: Merchant bankers manage and underwrite new issues, provide Consultancy and
Corporate advisory Services for corporate clients on raising funds and other financial aspects.

In India, merchant banking services are carried out by commercial banks.

The merchant banks are also called as Investment Company.

Merchant Banker has been defined under the Securities and Exchange Board of India (Merchant
Bankers) Rules, 1992 as, "any person who is engaged in the business of issue management either by
making arrangements regarding selling, buying or subscribing to securities as manager, consultant,
advisor or rendering corporate advisory service in relation to such issue management"

Q Source: Chapter 7: Shankar Ganesh: Indian Economy – Key Concepts

57 Consider the following statements about the office of President of India.


1. No person has occupied the office for more than two complete terms.
2. Every President has served the full term of the office.

Which of the above is/are correct?


A. 1 only
B. 2 only
C. Both 1 and 2
D. None

Correct Answer : A

Answer Justification :

Justification: Statement 1: Dr. Rajendra Prasad has served for two terms (1950-1962), and apart
from him none has served more than once. So, 1 is correct.

Statement 2: So far two Presidents, Dr Zakir Hussain and Fakhruddin Ali Ahmed, have died during
their term of office. So, 2 is incorrect.

For example, when President Dr Zakir Hussain died in May, 1969, the then Vice-President, VV Giri
was acting as the President.

43
https://telegram.me/UPSCMaterials https://telegram.me/FreeUPSCMaterials https://telegram.me/MaterialforExam
Mock Test 11

Soon after VV Giri resigned to contest the election of the President. Then the Chief Justice of India,
M Hidayatullah worked as the officiating President.

Q Source: Chapter 17: Indian Polity: M Laxmikanth

58 Consider the following statements about Cassini–Huygens.


1. It is an unmanned spacecraft sent to the planet Saturn.
2. Its design includes an orbiter for the concerned planet and a lander for the moon of that planet.

Which of the above is/are correct?


A. 1 only
B. 2 only
C. Both 1 and 2
D. None

Correct Answer : C

Answer Justification :

Justification: Launched in 1997, the Cassini mission — a cooperation between NASA, the
European Space Agency and the Italian Space Agency — has sent back thousands of stunning
images and made numerous discoveries about the ringed planet and its moons.

Cassini–Huygens is an unmanned spacecraft sent to the planet Saturn. Cassini is the fourth space
probe to visit Saturn and the first to enter orbit. Its design includes a Saturn orbiter and a lander
for the moon Titan. The lander, called Huygens, landed on Titan in 2005. The spacecraft was
launched on October 15, 1997. This was the first landing ever accomplished in the outer Solar
System.

Objectives of the mission:

Determine the three-dimensional structure and dynamic behavior of the rings of Saturn.

Determine the composition of the satellite surfaces and the geological history of each object.

Determine the nature and origin of the dark material on Iapetus’s leading hemisphere.

Measure the three-dimensional structure and dynamic behavior of the magnetosphere.

Study the dynamic behavior of Saturn’s atmosphere at cloud level.

Study the time variability of Titan’s clouds and hazes.

44
https://telegram.me/UPSCMaterials https://telegram.me/FreeUPSCMaterials https://telegram.me/MaterialforExam
Mock Test 11

Characterize Titan’s surface on a regional scale.

Q Source:
http://www.insightsonindia.com/2018/09/08/insights-daily-current-affairs-08-september-2018/

59 The policy most conducive to raising the general level of prices in the economy is

A. Loose monetary policy and loose fiscal policy


B. Loose monetary policy and tight fiscal policy
C. Tight monetary policy and tight fiscal policy
D. Tight monetary policy and loose fiscal policy

Correct Answer : A

Answer Justification :

Justification: Loose monetary policy means CRR, SLR, repo are lowered and banks now are willing
to lend more to the general public, who would spend it on goods ending up inflating their prices.

Loose fiscal policy means excess spending by the government which also pumps in money in the
economy leading to a rise in the general level of prices.

When both are loosened, the effect is pronounced. Tight policies do exactly the opposite.

Q Source: Chapter 9: Shankar Ganesh: Indian Economy – Key Concepts

60 India and the U.S. have signed the foundational or enabling agreement Communications
Compatibility and Security Agreement (COMCASA) on the side-lines of the inaugural 2+2 dialogue.
Consider the following about it.
1. It comes into force immediately, and as an experimental measure is valid only for the period of a
year.
2. COMCASA allows India to procure transfer specialised equipment for encrypted communications for
US origin military platforms.
3. The COMCASA, however, will not allow both sides to operate on the same communication systems
and is not an “interoperable” environment for militaries.

Select the correct answer using the codes below.


A. 1 and 2 only
B. 2 only
C. 1 and 3 only
D. 2 and 3 only

Correct Answer : B

45
https://telegram.me/UPSCMaterials https://telegram.me/FreeUPSCMaterials https://telegram.me/MaterialforExam
Mock Test 11

Answer Justification :

Justification: COMCASA is one of the four foundational agreements that the U.S. signs with allies
and close partners to facilitate interoperability between militaries and sale of high end technology.

S1: COMCASA is an India-specific version of the Communication and Information on Security


Memorandum of Agreement (CISMOA). It comes into force immediately, and is valid for a period 10
years.

S2: COMCASA allows India to procure transfer specialised equipment for encrypted
communications for US origin military platforms like the C-17, C-130 and P-8Is. Currently, these
platforms use commercially available communication systems.

COMCASA is a “technology enabler” to help transfer high-tech avionics, encrypted communication


and electronic systems to India as well as ensure secrecy of its C4ISR (command, control,
communications, computers, intelligence, surveillance and reconnaissance) systems from leaking to
other countries like Russia. This agreement would allow the interoperability of India and United
States equipments.

S3: The Comcasa will allow both sides to operate on the same communication systems, enabling an
“interoperable” environment for militaries. Without this agreement, the US cannot part with highly
coded communication equipment with the military platforms they sell to India. India has to depend
on commercially available less secure systems on, otherwise, high-end platforms like C-130Js and
the P8I maritime surveillance aircraft, among others.

COMCASA will change that. Now the US government will be able to give the go-ahead to install the
best communication equipment on relevant platforms such as aircraft being sold to India.

Learning: The Comcasa provides the legal framework for the US to part with its sensitive
communication equipment and codes to enable transfer of realtime operational information.

This equipment is largely used for ground-to-air communication, on installed US-origin


military aircraft, to enable best battle situation awareness.

The US data link is considered the most secure communication platform, which will also allow
India access to big data base of American intelligence, including real-time imagery.

COMCASA will effectively mean India sharing the real-time American intelligence on military
deployments by China and Pakistan.

COMCASA will also help India obtain the armed version of the Sea Guardian drones. The US
could not part with the weapon systems on the drone without COMCASA which will now allow
installation of data and communication systems.

Q Source: As mentioned above

46
https://telegram.me/UPSCMaterials https://telegram.me/FreeUPSCMaterials https://telegram.me/MaterialforExam
Mock Test 11

61 Consider the following statements with reference to the Public Account of India.
Assertion (A): This account is operated by executive action
Reason (R): All capital receipts of the government are credited to this account.

In the context of the above, which of these is correct?


A. A is correct, and R is an appropriate explanation of A.
B. A is correct, but R is not an appropriate explanation of A.
C. A is correct, but R is incorrect.
D. Both A and R are incorrect.

Correct Answer : C

Answer Justification :

Justification: All other public money (other than those which are credited to the Consolidated
Fund of India) received by or on behalf of the Government of India shall be credited to the Public
Account of India.

This includes provident fund deposits, judicial deposits, savings bank deposits, departmental
deposits, remittances and so on. This account is operated by executive action, that is, the payments
from this account can by made without parliamentary appropriation.

Such payments are mostly in the nature of banking transactions.

Q Source: Chapter - Parliament: Indian Polity: M Laxmikanth

62 Consider the following statements. The National Mission on Government eMarketplace (GeM)
1. Provides online, end to end solution for procurement of commonly used goods and services for all
Central Government Departments, State Governments and PSUs
2. Aims to promote inclusiveness by empowering various categories of sellers and service providers
such as MSMEs, start-ups, domestic manufacturers and Women SHGs

Which of the above is/are correct?


A. 1 only
B. 2 only
C. Both 1 and 2
D. None

Correct Answer : C

Answer Justification :

Justification: The government has launched the National Mission on Government eMarketplace
(GeM) for increasing awareness and accelerating the use of (GeM).

Highlights of the mission:

47
https://telegram.me/UPSCMaterials https://telegram.me/FreeUPSCMaterials https://telegram.me/MaterialforExam
Mock Test 11

The National Mission will cover all central government departments, states and public sector
undertakings in a mission mode.

It is aimed at creating awareness about GeM, train the buyers and sellers, get them
registered in GeM and increase the procurement through GeM.

It also aims to promote inclusiveness by empowering various categories of sellers and service
providers such as MSMEs, start-ups, domestic manufacturers, women entrepreneurs, and
Self-Help Groups.

It will also Highlight and communicate the value addition via transparency and efficiency in
public procurement, including corruption-free governance in sync with the Central
Government’s objective of ‘Minimum Government, Maximum Governance’.

The mission also aims to give a boost to cashless, contactless, paperless transactions in line
with Digital India objectives.

It aims to enhance transparency, efficiency and speed in public procurement of goods and
services and eliminate corruption.

Performance:

The e-marketplace completed two years in 2018 and on August 26, 2018, it crossed Rs 10,800 crore
in terms of value and 6.96 lakh in terms of volume of transactions through the platform.

The platform has more than 1.35 lakh sellers offering 4.43 lakh products and around 26,500
organisations as buyers.

Q Source:
http://www.insightsonindia.com/2018/09/06/insights-daily-current-affairs-06-september-2018/

63 Which of the following are key features of Cabinet Committees in India?


1. They are not mentioned in the Constitution and the Rules of Business of Council of Ministers.
2. They cannot take final decisions on matters referred to them by the Cabinet, which must be taken
by the Council of Ministers as a whole.

Which of the above is/are correct?


A. 1 only
B. 2 only
C. Both 1 and 2
D. None

48
https://telegram.me/UPSCMaterials https://telegram.me/FreeUPSCMaterials https://telegram.me/MaterialforExam
Mock Test 11

Correct Answer : D

Answer Justification :

Justification: Statement 1: They are extra-constitutional in emergence. In other words, they are
not mentioned in the Constitution. However, the Rules of Business provide for their establishment.

Statement 2: They not only sort out issues and formulate proposals for the consideration of the
Cabinet, but also take decisions. However, the Cabinet can review their decisions.

They are an organisational device to reduce the enormous workload of the Cabinet. They also
facilitate in-depth examination of policy issues and effective coordination.

Learning: They usually include only Cabinet Ministers. However, the non-cabinet Ministers are not
debarred from their membership.

They not only include the Ministers in charge of subjects covered by them but also include
other senior Ministers.

They are set up by the Prime Minister according to the exigencies of the time and
requirements of the situation. Hence, their number, nomenclature, and composition varies
from time to time.

They are mostly headed by the Prime Minister. Some times other Cabinet Ministers,
particularly the Home Minister or the Finance Minister, also acts as their Chairman.

But, in case the Prime Minister is a member of a committee, he invariably presides over it.

Q Source: Chapter – Cabinet Committees: Indian Polity: M Laxmikanth

64 Which of these are the members of the Cabinet Committee on Economic Affairs (CCEA)?
1. Minister of Home Affairs
2. Minister of External Affairs
3. Minister of Corporate Affairs
4. Minister of Defence

Select the correct answer using the codes below.


A. 1, 2 and 3 only
B. 1 and 3 only
C. 2 and 4 only
D. 1, 2, 3 and 4

Correct Answer : D

49
https://telegram.me/UPSCMaterials https://telegram.me/FreeUPSCMaterials https://telegram.me/MaterialforExam
Mock Test 11

Answer Justification :

Justification: Composition:

Prime Minister, Minister of Home Affairs, Minister of External Affairs, Minister of Finance;
and Minister of Corporate Affairs.

Minister of Urban Development; Minister of Housing and Urban Poverty Alleviation; and
Minister of Information and Broadcasting.

Minister of Road Transport and Highways; Minister of Shipping.

Minister of Defence; Minister of Railways

Minister of Chemicals and Fertilizers; and Minister of Parliamentary Affairs.

Minister of Law and Justice; and Minister of Electronics and Information Technology.

Minister of Civil Aviation.

Minister of Food Processing Industries.

Minister of Agriculture and Farmers Welfare

Q Source: http://cabsec.nic.in/cabinetcommittees.php

65 Consider the following statements.


1. South Asian Women Development Forum (SAWDF) is first SAARC Recognized Body organisation
working on women entrepreneur’s issues.
2. The International Women's Rights Action Watch (IWRAW) was organized to promote recognition of
women’s human rights under the United Nations Convention on the Elimination of All Forms of
Discrimination against Women (CEDAW),

Which of the above is/are correct?


A. 1 only
B. 2 only
C. Both 1 and 2
D. None

50
https://telegram.me/UPSCMaterials https://telegram.me/FreeUPSCMaterials https://telegram.me/MaterialforExam
Mock Test 11

Correct Answer : C

Answer Justification :

Justification: Statement 1: SAWDF is an autonomous, nonprofit organization based in Kathmandu,


Nepal.

It was granted the status of SAARC Recognized Body by 36th Session of SAARC Council Ministers
at 18th SAARC Summit held in Kathmandu in 2014.

It is first SAARC Recognized Body organisation working on women entrepreneurs issues.

It represents national women entrepreneurs’ associations or organizations, institutions working


towards economic issues of women entrepreneurs in SAARC Region.

Its membership is based on organizations with extensive work around women’s entrepreneurship.

Statement 2: The International Women's Rights Action Watch (IWRAW) was organized in 1985 at
the Third World Conference on Women in Nairobi, Kenya, to promote recognition of women’s
human rights under the United Nations Convention on the Elimination of All Forms of
Discrimination against Women (CEDAW), an international human rights treaty.

IWRAW was founded on the belief that the human rights of women and girls are essential to
development and that equality between women and men will only be achieved through use of
international human rights principles and processes.

Since its inception, IWRAW’s program has expanded to encompass advocacy for women’s human
rights under all the international human rights treaties

Q Source: http://hrlibrary.umn.edu/iwraw/

66 Debt Recovery Tribunals were established to facilitate the debt recovery involving banks and other
financial institutions with their customers. Consider the following about DRTs.
1. DRTs are informal institutions established on an ad-hoc basis following natural principles of justice.
2. Appeals against orders passed by DRTs lie only before the High Courts or the Supreme Court.
3. A DRT is presided by a nominee of the concerned bank or financial institution.

Select the correct answer using the codes below.


A. 1 only
B. 2 and 3 only
C. None of the above
D. 2 only

Correct Answer : C

Answer Justification :

Justification: DRTs were set up after the passing of Recovery of Debts due to Banks and Financial

51
https://telegram.me/UPSCMaterials https://telegram.me/FreeUPSCMaterials https://telegram.me/MaterialforExam
Mock Test 11

Institutions Act (RDBBFI), 1993.

Section 3 of the RDDBFI Act empowers the Central government to establish DRTs.

Appeals against orders passed by DRTs lie before Debts Recovery Appellate Tribunal (DRAT).

Powers and functions:

The Debts Recovery Tribunal (DRT) enforces provisions of the Recovery of Debts Due to Banks and
Financial Institutions (RDDBFI) Act, 1993 and also Securitization and Reconstruction of Financial
Assets and Enforcement of Security Interests (SARFAESI) Act, 2002.

The Debts Recovery Tribunal (DRT) are fully empowered to pass comprehensive orders and can
travel beyond the Civil procedure Code to render complete justice. A Debts Recovery Tribunal
(DRT) can hear cross suits, counter claims and allow set offs.

However, a Debts Recovery Tribunal (DRT) cannot hear claims of damages or deficiency of services
or breach of contract or criminal negligence on the part of the lenders. In addition, a Debts
Recovery Tribunal (DRT) cannot express an opinion beyond its domain, or the list pending before it.

The Debts Recovery Tribunal can appoint Receivers, Commissioners, pass ex-parte orders, ad-
interim orders, interim orders apart from powers to Review its own decisions and hear appeals
against orders passed by the Recovery Officers of the Tribunal.

Other key facts:

Statement 3: A DRT is presided over by a presiding officer who is appointed by the central govt. and
who shall be qualified to be a District Judge; with tenure of 5 years or the age of 62, whichever is
earlier.

No court in the country other than the SC and the HCs and that too, only under articles 226 and
227 of the Constitution have jurisdiction over this matter.

Q Source:
http://www.insightsonindia.com/2018/09/10/insights-daily-current-affairs-10-september-2018/

67 Consider the following statements.


1. The Speaker is elected by the Lok Sabha from amongst its members.
2. The date of election of the Speaker is fixed by the President.
3. Whenever the Lok Sabha is dissolved, the Speaker does not vacate his office and continues till the
newly elected Lok Sabha meets.

Select the correct answer using the codes below.


A. 3 only
B. 2 and 3 only
C. 1, 2 and 3
D. 2 only

52
https://telegram.me/UPSCMaterials https://telegram.me/FreeUPSCMaterials https://telegram.me/MaterialforExam
Mock Test 11

Correct Answer : C

Answer Justification :

Justification: Statement 1 and 2: The Speaker is elected by the Lok Sabha from amongst its
members (as soon as may be, after its first sitting). Whenever the office of the Speaker falls vacant,
the Lok Sabha elects another member to fill the vacancy. The date of election of the Speaker is fixed
by the President.

Learning: When a resolution for the removal of the Speaker is under consideration of the House,
he cannot preside at the sitting of the House, though he may be present.

However, he can speak and take part in the proceedings of the House at such a time and vote in the
first instance, though not in the case of an equality of votes.

It should be noted here that, whenever the Lok Sabha is dissolved, the Speaker does not vacate his
office and continues till the newly elected Lok Sabha meets.

Q Source: Chapter – Parliament: Indian Polity: M Laxmikanth

68 Which of the following authorities adjourns the Lok Sabha House or suspends the meeting in absence
of a quorum?

A. President of India
B. Speaker, Lok Sabha
C. Leader of the House
D. Secretary, Lok Sabha

Correct Answer : B

Answer Justification :

Learning: The speaker has the following powers and duties:

He maintains order and decorum in the House for conducting its business and regulating its
proceedings. This is his primary responsibility and he has final power in this regard.

He is the final interpreter of the provisions of (a) the Constitution of India, (b) the Rules of
Procedure and Conduct of Business of Lok Sabha, and (c) the parliamentary precedents,
within the House.

He adjourns the House or suspends the meeting in absence of a quorum. The quorum to
constitute a meeting of the House is one-tenth of the total strength of the House.

53
https://telegram.me/UPSCMaterials https://telegram.me/FreeUPSCMaterials https://telegram.me/MaterialforExam
Mock Test 11

Q Source: Chapter – Parliament: Indian Polity: M Laxmikanth

69 Consider the following statements. As a consequence of sterilization


1. A commercial bank’s total reserves and deposits remain unchanged.
2. RBI’s foreign exchange holding goes up.
3. There is an increase in the stock of high powered money.

Select the correct answer using the codes below.


A. 2 only
B. 1 and 3 only
C. 2 and 3 only
D. 1, 2 and 3

Correct Answer : D

Answer Justification :

Concept and Justification: RBI often uses its instruments of money creation for stabilising the
stock of money in the economy from external shocks.

Suppose due to future growth prospects in India investors from across the world increase their
investments in Indian bonds which under such circumstances, are likely to yield a high rate of
return. They will buy these bonds with foreign currency.

Since one cannot purchase goods in the domestic market with foreign currency, a person or a
financial institution who sells these bonds to foreign investors will exchange its foreign currency
holding into rupee at a commercial bank.

The bank, in turn, will submit this foreign currency to RBI and its deposits with RBI will be credited
with equivalent sum of money. What kind of adjustments take place from this entire transaction?

Statement 1: The commercial bank’s total reserves and deposits remain unchanged (it has
purchased the foreign currency from the seller using its vault cash, which, therefore, goes down;
but the bank’s deposit with RBI goes up by an equivalent amount – leaving its total reserves
unchanged).

Statement 2: There will, however, be increments in the assets and liabilities on the RBI balance
sheet. RBI’s foreign exchange holding goes up. On the other hand, the deposits of commercial banks
with RBI also increase by an equal amount. But that means an increase in the stock of high powered
money – which, by definition, is equal to the total liability of RBI. With money multiplier in
operation, this, in turn, will result in increased money supply in the economy.

Q Source: Page 44: Chapter 3: 12th NCERT: Macroeconomics

70 Consider the following statements.


1. The Chairman of the Rajya Sabha can be removed from his office only if he is removed from the

54
https://telegram.me/UPSCMaterials https://telegram.me/FreeUPSCMaterials https://telegram.me/MaterialforExam
Mock Test 11

office of the Vice-President.


2. Unlike the Speaker, the Chairman of Rajya Sabha is not a member of the House.

Which of the above is/are correct?


A. 1 only
B. 2 only
C. Both 1 and 2
D. None

Correct Answer : C

Answer Justification :

Justification: Statement 1 and 2: The vice-president of India is the ex-officio Chairman of the Rajya
Sabha.

If he is to be removed from the Chairman of the house, he must be impeached as the Vice-President.

This is unlike the institution of the speaker, where the Speaker is a member of the house and can be
removed by the Lok Sabha.

Q Source: Chapter – Parliament: Indian Polity: M Laxmikanth

71 The Leader of Lok Sabha is

A. Appointed by the Presiding Officer


B. Elected by the House
C. Nominated by the President of India
D. None of the above

Correct Answer : D

Answer Justification :

Learning: Under the Rules of Lok Sabha, the ‘Leader of the House’ means the prime minister, if he
is a member of the Lok Sabha, or a minister who is a member of the Lok Sabha and is nominated by
the prime minister to function as the Leader of the House.

There is also a ‘Leader of the House’ in the Rajya Sabha. He is a minister and a member of the
Rajya Sabha and is nominated by the prime minister to function as such.

The leader of the house in either House is an important functionary and exercises direct influence
on the conduct of business. He can also nominate a deputy leader of the House.

Q Source: Chapter – Parliament: Indian Polity: M Laxmikanth

55
https://telegram.me/UPSCMaterials https://telegram.me/FreeUPSCMaterials https://telegram.me/MaterialforExam
Mock Test 11

72 Barter exchange would work the best or most efficiently in

A. A closed small economy


B. An open large economy
C. A closed large economy
D. An open small economy

Correct Answer : A

Answer Justification :

Justification: An open economy is the one that engages in international trade, and a closed
economy does not.

Barter exchanges become extremely difficult in a large economy because of the high costs people
would have to incur looking for suitable persons to exchange their surpluses.

Option D: When the large economy is also open, even though the possibilities of finding a barter
exchange increases, the costs and information involved in such an exchange hinders its possibilities.

Therefore, the foremost role of money is that it acts as a medium of exchange which can be
accepted universally.
th
Q Source: Chapter 3: 12 NCERT: Macroeconomics

73 When the Lok Sabha is dissolved, which of these bills do NOT lapse?
#000000
1. A bill pending in the Rajya Sabha but not passed by the Lok Sabha
2. A bill passed by the Lok Sabha but pending in the Rajya Sabha
3. A bill passed by both Houses but returned by the president for reconsideration of Houses

Select the correct answer using the codes below.


A. 1 and 2 only
B. 1 and 3 only
C. 2 and 3 only
D. 1, 2 and 3

Correct Answer : B

Answer Justification :

Justification: On dissolution of Lok Sabha, all business including bills, motions, resolutions,
notices, petitions and so on pending before it or its committees lapse. They (to be pursued further)
must be reintroduced in the newly-constituted Lok Sabha.

However, some pending bills and all pending assurances that are to be examined by the Committee

56
https://telegram.me/UPSCMaterials https://telegram.me/FreeUPSCMaterials https://telegram.me/MaterialforExam
Mock Test 11

on Government Assurances do not lapse on the dissolution of the Lok Sabha. The position with
respect to lapsing of bills is as follows:

A bill pending in the Lok Sabha lapses (whether originating in the Lok Sabha or transmitted
to it by the Rajya Sabha).

A bill passed by the Lok Sabha but pending in the Rajya Sabha lapses.

A bill not passed by the two Houses due to disagreement and if the president has notified the
holding of a joint sitting before the dissolution of Lok Sabha, does not lapse.

A bill pending in the Rajya Sabha but not passed by the Lok Sabha does not lapse.

A bill passed by both Houses but pending assent of the president does not lapse.

A bill passed by both Houses but returned by the president for reconsideration of Houses does
not lapse.

Q Source: Chapter – Parliament: Indian Polity: M Laxmikanth

74 In addition to the members of a House, who has the right to speak and take part in the proceedings of
either House of the Parliament or any of its committees?
1. Any Union Minister
2. Attorney general of India
3. Cabinet Secretary

Select the correct answer using the codes below.


A. 1 and 2 only
B. 2 and 3 only
C. 1 only
D. 1, 2 and 3

Correct Answer : A

Answer Justification :

Justification: Every minister and the attorney general of India have the right to speak and take
part in the proceedings of either House, any joint sitting of both the Houses and any committee of
Parliament of which he is a member, without being entitled to vote.

There are two reasons underlying this constitutional provision:

57
https://telegram.me/UPSCMaterials https://telegram.me/FreeUPSCMaterials https://telegram.me/MaterialforExam
Mock Test 11

A minister can participate in the proceedings of a House, of which he is not a member. In


other words, a minister belonging to the Lok Sabha can participate in the proceedings of the
Rajya Sabha and vice-versa.

A minister, who is not a member of either House, can participate in the proceedings of both
the Houses.

Q Source: Chapter – Parliament: Indian Polity: M Laxmikanth

75 The velocity of circulation of money is likely to increase with

A. Decline in Gross Domestic Product for the same amount of money supply in the economy
B. Greater use of credit cards
C. Severe reduction in employment in an economy with a steadily decreasing population
D. Greater use of cash as against online banking

Correct Answer : B

Answer Justification :

Background: You may perhaps find it surprising that an economy can use a money balance worth
only Rs 100 for making transactions worth Rs 200 per month. The answer to this riddle is simple –
each rupee is changing hands twice a month.

On the first day, it is being transferred from the employer’s pocket to that of the worker and
sometime during the month, it is passing from the worker’s hand to the employer’s. The number of
times a unit of money changes hands during the unit period is called the velocity of circulation.

Velocity of circulation is measured by dividing GDP by the country's total money supply. A high
velocity of circulation in a country indicates a high degree of inflation. It helps in determining how
vigorous a country's economy is.

If the velocity of money is increasing, then transactions are occurring between individuals more
frequently

Justification: Option B: Credit card decreases the money people are willing to hold, and thus
increase the money velocity.

Also, it reduces the household’s need to hold cash and create debt against a given limited money
supply. The creation of debt, a powerful tool behind credit creation, is responsible for increasing the
transactions between individuals.

In simpler terms, if you hold all your money and spend nothing, the velocity of money with respect
to you is zero.

If you hold less cash, and spend more, the velocity is more.

58
https://telegram.me/UPSCMaterials https://telegram.me/FreeUPSCMaterials https://telegram.me/MaterialforExam
Mock Test 11

But, if you do not hold any cash and take debt (using credit cards) to buy things, the velocity of
money created by you is very high. Remember that, its all about how much the money is rotated
depends on how much of it is exchanged how frequently with how many people.

Option A, C and D all go against the logic mentioned right above.

Q Source: Page 35: Chapter 3: 12th NCERT: Macroeconomics

76 Consider the following with reference to Adjournment Motion introduced in the Parliament.
Assertion (A): Rajya Sabha is not permitted to make use of this device.
Reason (R): It involves an element of censure against the government.

In the context of the above, which of these is correct?


A. A is correct, and R is an appropriate explanation of A.
B. A is correct, but R is not an appropriate explanation of A.
C. A is correct, but R is incorrect.
D. Both A and R are incorrect.

Correct Answer : A

Answer Justification :

Justification: It is introduced in the Parliament to draw attention of the House to a definite matter
of urgent public importance, and needs the support of 50 members to be admitted.

As it interrupts the normal business of the House, it is regarded as an extraordinary device.

It involves an element of censure against the government and hence Rajya Sabha is not permitted to
make use of this device.

The discussion on an adjournment motion should last for not less than two hours and thirty minutes.

The right to move a motion for an adjournment of the business of the House is subject to several
restrictions which you can find at the Q Source.

Q Source: Chapter – Parliament: Indian Polity: M Laxmikanth

77 Phillips curve shows the relationship between

A. Headline and core Inflation


B. Inflation and tax revenues
C. Tax revenues and population cohort
D. Inflation and unemployment

Correct Answer : D

59
https://telegram.me/UPSCMaterials https://telegram.me/FreeUPSCMaterials https://telegram.me/MaterialforExam
Mock Test 11

Answer Justification :

Justification: The Phillips curve is an economic concept developed by A. W. Phillips stating that
inflation and unemployment have a stable and inverse relationship. The theory claims that with
economic growth comes inflation, which in turn should lead to more jobs and less unemployment.
However, the original concept has been somewhat disproven empirically due to the occurrence of
stagflation in the 1970s, when there were high levels of both inflation and unemployment.

Q Source: Chapter 9: Shankar Ganesh: Indian Economy – Key Concepts

78 Which of the following is NOT considered a part of money supply in India?


1. Currency notes
2. Currency coins
3. Current account deposits

Select the correct answer using the codes below.


A. 1 and 2 only
B. 2 only
C. 3 only
D. All of the above are part of the money supply.

Correct Answer : D

Answer Justification :

Justification: In a modern economy money consists mainly of currency notes and coins issued by
the monetary authority of the country. In India currency notes are issued by the Reserve Bank of
India (RBI), which is the monetary authority in India.

60
https://telegram.me/UPSCMaterials https://telegram.me/FreeUPSCMaterials https://telegram.me/MaterialforExam
Mock Test 11

However, coins are issued by the Government of India. Apart from currency notes and coins, the
balance in savings, or current account deposits, held by the public in commercial banks is also
considered money (or a part of the money supply) since cheques drawn on these accounts are used
to settle transactions. Such deposits are called demand deposits as they are payable by the bank on
demand from the accountholder. Other deposits, e.g. fixed deposits, have a fixed period to maturity
and are referred.

Q Source: Page 38: Chapter 3: 12th NCERT: Macroeconomics

79 Who is authorized to certify whether a bill is an ordinary bill or a money bill?

A. President of India
B. Speaker, Lok Sabha
C. Chairman, Rajya Sabha
D. Leader of the House

Correct Answer : B

Answer Justification :

Justification: If any question arises whether a bill is a money bill or not, the decision of the
Speaker of the Lok Sabha is final.

His decision in this regard cannot be questioned in any court of law or in the either House of
Parliament or even the president.

When a money bill is transmitted to the Rajya Sabha for recommendation and presented to the
president for assent, the Speaker endorses it as a money bill.

Q Source: Chapter – Parliament: Indian Polity: M Laxmikanth

80 Joint sitting is called by the President in which of the following situations?


1. If a bill passed on one house is rejected by the other House
2. If more than six months have elapsed from the date of the receipt of the bill by the other House

Which of the above is/are correct?


A. 1 only
B. 2 only
C. Both 1 and 2
D. None

Correct Answer : C

Answer Justification :

61
https://telegram.me/UPSCMaterials https://telegram.me/FreeUPSCMaterials https://telegram.me/MaterialforExam
Mock Test 11

Justification: It is extraordinary machinery provided by the Constitution to resolve a deadlock


between the two Houses over the passage of a bill.

A deadlock is deemed to have taken place under any one of the following three situations after a bill
has been passed by one House and transmitted to the other House:

if the bill is rejected by the other House;

if the Houses have finally disagreed as to the amendments to be made in the bill; or

if more than six months have elapsed from the date of the receipt of the bill by the other
House without the bill being passed by it.

Learning: In the above three situations, the president can summon both the Houses to meet in a
joint sitting for the purpose of deliberating and voting on the bill.

It must be noted here that the provision of joint sitting is applicable to ordinary bills or
financial bills only and not to money bills or Constitutional amendment bills.

In the case of a money bill, the Lok Sabha has overriding powers, while a Constitutional
amendment bill must be passed by each House separately.

The Speaker of Lok Sabha presides over a joint sitting of the two Houses and the Deputy
Speaker, in his absence.

Q Source: Chapter – Parliament: Indian Polity: M Laxmikanth

81 Credit creation will NOT take place

A. If banks reduce their emergency reserves to zero


B. If Non-Banking Financial Institutions (NBFIs) start lending to consumers
C. If there is no borrowing and lending by the corporate sector
D. None of the above

Correct Answer : D

Answer Justification :

Background: Banks do not keep cent per cent reserve against deposits in order to meet the
demands of depositors. The bank is not a cloak room where you can keep your currency notes or
coins and claim those very notes or coins back when you desire. It is generally understood that

62
https://telegram.me/UPSCMaterials https://telegram.me/FreeUPSCMaterials https://telegram.me/MaterialforExam
Mock Test 11

money received by the bank is meant to be advanced to others. A depositor has to be content simply
with the bank’s promise or undertaking to pay him back whenever he makes a demand.

Justification: The bank is thus enabled to erect a vast superstructure of credit on the basis of a
small cash reserve. The bank is able to lend money and charge interest without parting with cash.

Similarly, the bank buys securities and pays the seller with its own cheque which again is no cash; it
is just a promise to pay cash. The cheque is deposited in some bank and a deposit is created or
credit is created for the seller of the securities. This is credit creation.

Thus, term ‘credit creation’ implies a situation when a bank may receive interest simply by
permitting customers to overdraw their accounts or by purchasing securities and paying for them
with its own cheques, thus increasing the total bank deposits.

Q Source: Page 42: Chapter 3: 12th NCERT: Macroeconomics

82 Vote of Credit is given by the Parliament to the executive when

A. Additional expenditure upon some new service not contemplated in the budget for that year is
sought
B. Money has been spent on any service during a financial year in excess of the amount granted
for that service in the budget for that year
C. Meeting an unexpected demand upon the resources of India which cannot be stated with the
details ordinarily given in a budget
D. Executive seeks advance appropriation before the examination and enactment of the budget
to meet current expenditures

Correct Answer : C

Answer Justification :

Justification & Learning: The above options refer to different kinds of grants that are given to
executive apart from the normal annual budget.

Additional Grant: It is granted when a need has arisen during the current financial year for
additional expenditure upon some new service not contemplated in the budget for that year.

Excess Grant: It is granted when money has been spent on any service during a financial year in
excess of the amount granted for that service in the budget for that year. It is voted by the Lok
Sabha after the financial year.

Vote of Credit: It is granted for meeting an unexpected demand upon the resources of India, when
on account of the magnitude or the indefinite character of the service, the demand cannot be stated
with the details ordinarily given in a budget. Hence, it is like a blank cheque given to the Executive
by the Lok Sabha.

Exceptional Grant: It is granted for a special purpose and forms no part of the current service of any

63
https://telegram.me/UPSCMaterials https://telegram.me/FreeUPSCMaterials https://telegram.me/MaterialforExam
Mock Test 11

financial year.

Option D is about Vote on Credit, which will be covered in later tests.

Q Source: Chapter – Parliament: Indian Polity: M Laxmikanth

83 Which of the following statements about the Public Accounts Committee (PAC) is INCORRECT?

A. It was first setup under the provisions of the Government of India Act of 1919.
B. Its members are drawn from both houses of the Parliament.
C. A minister cannot be elected as a member of the committee.
D. Chairman of the Committee is appointed by the Leader of the House.

Correct Answer : D

Answer Justification :

Learning: This committee was setup first in 1921 under the provisions of the Government of India
Act of 1919 and has since been in existence. So, A is correct.

At present, it consists of 22 members (15 from the Lok Sabha and 7 from the Rajya Sabha). The
members are elected by the Parliament every year from amongst its members according to the
principle of proportional representation by means of the single transferable vote. So, B is correct.

A minister cannot be elected as a member of the committee. The chairman of the committee is
appointed by the Speaker from amongst its members. So, D is incorrect.

The function of the committee is to examine the annual audit reports of the comptroller and auditor
general of India (CAG), which are laid before the Parliament by the president.

Q Source: Chapter – Parliament: Indian Polity: M Laxmikanth

84 The Reserve Bank of India (RBI), primarily, plays which of the following roles in the Indian economy?
1. Control of credit creation
2. Financing of budget deficits

Which of the above is/are correct?


A. 1 only
B. 2 only
C. Both 1 and 2
D. None

Correct Answer : A

Answer Justification :

64
https://telegram.me/UPSCMaterials https://telegram.me/FreeUPSCMaterials https://telegram.me/MaterialforExam
Mock Test 11

Justification: Statement 1: This concept has been covered in other questions. Control of money
supply and reserve deposit ration of banks clearly (logically) implies a control of credit creation.

Statement 2: This is not a primary function of the RBI, in fact this was banned after the passing of
the FRBM Act, 2003 because the ruling parties were leaning towards RBI to fund their expansionist
and populist budgets through printing of fresh currency by RBI.

The government has no legal authority to issue currency to fund its deficit. So it borrows money
either from the domestic or international market or sells treasury bills or government securities to
RBI, which issues currency to the government in return.

The government then pays for its expenses with this money. The money thus ultimately comes into
the hands of the general public (in the form of salary or sales proceeds of defense items etc.) and
becomes a part of the money supply. Financing of budget deficits by the governments in this fashion
is called Deficit Financing through Central Bank Borrowing.

Q Source: Page 43: Chapter 3: 12th NCERT: Macroeconomics

85 The “Base Effect”, when measuring inflation, refers to the impact of

A. Stock of domestic resources on inflation


B. Technological breakthroughs on inflation
C. The price level of last year on inflation measurement
D. Revision of consumption basket on inflation

Correct Answer : C

Answer Justification :

Learning: The base effect refers to the impact of the rise in price level (i.e. last year’s inflation) in
the previous year over the corresponding rise in price levels in the current year (i.e., current
inflation).

If the price index had risen at a high rate in the corresponding period of the previous year leading
to a high inflation rate, some of the potential rise is already factored in, therefore a similar absolute
increase in the Price index in the current year will lead to a relatively lower inflation rates.

On the other hand, if the inflation rate was too low in the corresponding period of the previous year,
even a relatively smaller rise in the Price Index will arithmetically give a high rate of current
inflation.

Q Source: Chapter 9: Shankar Ganesh: Indian Economy – Key Concepts

86 Under the ‘Minimum Reserve System’ followed by India


1. The RBI has to keep a minimum reserve value comprising of gold coin, gold bullion and foreign
currencies to issue fresh currency
2. The RBI cannot issue unlimited amount of currency by keeping this reserve and must allow for

65
https://telegram.me/UPSCMaterials https://telegram.me/FreeUPSCMaterials https://telegram.me/MaterialforExam
Mock Test 11

proportionate increase in this reserve with an expansion of currency base.

Which of the above is/are correct?


A. 1 only
B. 2 only
C. Both 1 and 2
D. None

Correct Answer : A

Answer Justification :

Justification: The relationship between note issue and its reserve backing is usually done on the
basis of a reserve system by central banks across the world. The reserve system provides guidelines
for the issue of new currencies.

In India, currencies are issued by the RBI with the backing of reserves comprised of gold and
foreign exchange (foreign currencies). For the issue of currencies, the RBI follows Minimum
Reserve System at present. The Minimum Reserve System (MRS) is followed from 1956 onwards.

Previously, this system was called as “proportional reserve system” where the amount of currency
issued had to be backed up by an equivalent or proportional amount of reserves.

But, the minimum reserve is a token of confidence and doesn’t have any practical connection with
amount new currencies issued by the RBI. Under the Minimum Reserve System, RBI can issue
unlimited amount of currency by keeping the reserve. But RBI follows some principle or rule for
issuing new currencies based upon economic growth and transaction needs of the people.

Under the Minimum Reserve System, the RBI has to keep a minimum reserve of Rs 200 crore
comprising of gold coin and gold bullion and foreign currencies. Out of the total Rs 200 crores,
Rs115 crore should be in the form of gold coins or gold bullion. The purpose of shifting to MRS was
to expand money supply to meet the needs of increasing transactions in the economy.

Q Source: Additional Research: Page 92: Shankar Ganesh: Indian Economy – Key Concepts

87 What do you understand by a ‘deflationary gap’ in the economy?

A. Low Inflation that encourages investments


B. Excess supply of goods and services with respect to the weak demand in the economy
C. Consistent decline in inflation at an exponential rate
D. Underproduction with respect to full employment GDP of the economy

Correct Answer : D

Answer Justification :

66
https://telegram.me/UPSCMaterials https://telegram.me/FreeUPSCMaterials https://telegram.me/MaterialforExam
Mock Test 11

Learning: Deflationary gap is the amount by which actual aggregate demand falls short of
aggregate supply at level of full employment.

It is a measure of amount of deficiency of aggregate demand. Deflationary gap causes a decline in


output, income and employment along with persistent fall in prices.

On the other hand, an inflationary gap is a signal that the economy is in the boom part of the trade
cycle, resources are being used over their capacity, factories are operating with increasing average
costs; wage rates increase because labour is used beyond normal hours at overtime pay rates.

Q Source: Additional Research: Chapter 9: Shankar Ganesh: Indian Economy – Key Concepts

88 “Expectations augmented Phillips curve” is often used to predict the short-term and long-term impact
of macroeconomic management policies in an economy. The term “expectations” denote expectations
about

A. GDP growth rate


B. Inflation
C. External balance
D. Environmental degradation

Correct Answer : B

Answer Justification :

Justification & Learning: As per the Phillips curve there is a ‘trade off’ between inflation and
unemployment i.e. an inverse relationship between them.

The curve suggests that lower the inflation, higher the unemployment and higher the inflation,
lower the unemployment.

However, the economist Milton Friedman argued that in the long run, there was no trade-off
between unemployment and inflation.

Suppose if government increases aggregate demand (AD) by monetary expansion, this not only
increases the AD but also increases the expectation that inflation will go up. As people expect
inflation to go up, they demand higher wages and unemployment returns to the initial level.

The expectations-augmented Phillips curve thus introduces adaptive expectations into the Phillips
curve and makes it more dynamic.

Q Source: Additional Research: Chapter 9: Shankar Ganesh: Indian Economy – Key Concepts

89 Which of the following are some of the main functions of the Office of Economic Adviser, Ministry of
Commerce and Industry, Government of India?
1. Analysis of trends of industrial production and growth
2. Planning and Gender Budgeting on behalf of the Department of Industrial Policy and Promotion

67
https://telegram.me/UPSCMaterials https://telegram.me/FreeUPSCMaterials https://telegram.me/MaterialforExam
Mock Test 11

3. Compiling and releasing monthly Wholesale Price Indices and Index of Core Industries Production

Select the correct answer using the codes below.


A. 1 and 2 only
B. 1 only
C. 2 and 3 only
D. 1, 2 and 3

Correct Answer : D

Answer Justification :

Learning: Office of the Economic Adviser (OEA) is an attached office of the Ministry of Commerce
& Industry.

The main functions include (apart from the ones mentioned in the question):

Policy Functions:

Economic policy inputs on industrial development.

Rendering advice relating to formulation of Industrial Policy, Foreign Trade Policy with
respect to industrial sector in general with thrust on manufacturing, issues relating to
bilateral and multilateral trade, as well as taxes and duties related to industry, including but
not restricted to safeguard and anti-dumping duties.

Examination of multilateral and bilateral issues and processing Policy Notes with economic
implications referred to the Office.

Statistical Functions

Developing other Indices on experimental basis, e.g. select business service price indices

Supervising as a ‘source agency’, compilation of monthly production statistics for identified


industrial items, their validation, and onward transmission for computation of the monthly
Index of Industrial Production (IIP) by Central Statistics Office.

Monthly Statistical compilation of macro indicators (secondary information).

Q Source: Additional Research: Chapter 9: Shankar Ganesh: Indian Economy – Key Concepts

68
https://telegram.me/UPSCMaterials https://telegram.me/FreeUPSCMaterials https://telegram.me/MaterialforExam
Mock Test 11

90 Consider the following about Capital to Risk Weighted Asset Ratio (CRAR).
1. RBI uses this ratio to ensure banks do not lend beyond their capacity.
2. This ratio is fixed under the Basel norms.

Which of the above is/are correct?


A. 1 only
B. 2 only
C. Both 1 and 2
D. None

Correct Answer : C

Answer Justification :

Justification: Capital Adequacy Ratio (CAR) or Capital to Risk Weighted Assets (CRAR) is the ratio
of a bank’s capital in relation to its risk weighted assets and current liabilities. It is decided by
central banks and bank regulators to prevent commercial banks from taking excess leverage and
becoming insolvent in the process.

It is measured as

Capital Adequacy Ratio = (Tier I + Tier II + Tier III (Capital funds)) /Risk weighted assets

The risk weighted assets take into account credit risk, market risk and operational risk.

The Basel III norms stipulated a capital to risk weighted assets of 8%. However, RBI norms can
stipulate a different norm for Indian scheduled commercial banks more than this CAR.

Q Source: Page 96: Shankar Ganesh: Indian Economy – Key Concepts

91 Traversing through the Nilgiri biosphere reserve, you are likely to find which of these key native
tribal groups?
1. Todas
2. Allar
3. Kora
4. Chik Baraik

Select the correct answer using the codes below.


A. 1 and 2 only
B. 3 and 4 only
C. 1 and 4 only
D. 2 and 3 only

Correct Answer : A

Answer Justification :

69
https://telegram.me/UPSCMaterials https://telegram.me/FreeUPSCMaterials https://telegram.me/MaterialforExam
Mock Test 11

Justification: Statement 1 and 2: Tribal groups like the Todas, Kotas, Irullas, Kurumbas, Paniyas,
Adiyans, Edanadan Chettis, Cholanaickens, Allar,Malayan, etc., are native to the reserve.

Statement 3: They were one of the ten indigenous tribes of the Great Andamanese people, originally
living on the eastern part of North Andaman Island in the Indian Ocean.

The tribe is now extinct, although some of the remaining Great Andamanese on Strait Island claim
to have Kora ancestors.

Statement 4: Chik Baraik is one of the caste of Chhattisgarh found in parts of Bihar, Jharkhand and
West Bengal, Port Blair and Orissa. Chik Baraiks were known for best fabric designers for their
kings and now weave clothes for their living.

Q Source: Revision: Previous test syllabus: Improvised UPSC CSP questions

92 When the aggregate demand in the economy is low during a recession that it is inadequate to create
sufficient number of jobs, this situation is called

A. Frictional unemployment
B. Structural unemployment
C. Disguised unemployment
D. Cyclical unemployment

Correct Answer : D

Answer Justification :

Learning: Frictional unemployment occurs because of the normal turnover in the labor market and
the time it takes for workers to find new jobs. Throughout the course of the year in the labor
market, some workers change jobs. So, A is wrong.

Structural unemployment occurs because of an absence of demand for a certain type of worker.
This typically happens when there are mismatches between the skills employers want and the skills
workers have. So, B is wrong.

We have already covered disguised unemployment.

Cyclical unemployment deals with an economy's business cycle. Cyclical unemployment occurs
when there are job losses during downturns and contractions in the business cycle. A lack of
demand is one of the main factors that cause cyclical unemployment.

Q Source: Revision: Previous test syllabus: Improvised UPSC CSP questions

93 As per the Reserve Bank of India Act, 1934, banks in India are classified into scheduled and non-
scheduled banks. Scheduled banks are those that have
1. Entered into the second schedule of the RBI Act, 1934
2. A capital base of at least Rs. 100 crore

70

You might also like